Sei sulla pagina 1di 56

r9l: t lslr 1)ll)vud

p.raqleoB e ssed lq8gu auo sllH aqr uo lno


'a8e1g,r aql urory deme 'cpsual)erer{l
sll'puelsr ar{l uo sprrq /![aJ pue sP
aJa,/vr aler{l leql osle lueaur ralezrt uado 1o

-uralsrr Jo alsel aql pqII qrnur ra^au


aql'saqrnbqu€ ou araM arar{I'lsaroJ-aw
s€M lsal aql lnq'ls€o) tpJou aq1;o sadols
uop€^Illn) a)€Jral;o saqcled rvra1 e prrp
,{,razrps auros aJaM araq; 'ut3.nn se1vt'lI Jo sa
dr+D IIe 'puel$ aqr yo dpoq aql'trpde eBE
pue /Ioor{DS r{sq8ug ue uerD Jaldaals reJ Sulqlawos 'erud1apelq6 latoH aql'loor{rs uor-ig p
paw8eufi p€q I '{Jor*'ryou.'1lort ol arueJ auo alar{1v\
luawalDas puad pasodu4-Jlas Jo lros e se-1r papre8a'r 1 se '1de dlensr,L lsnf ueql aroru ssaua{Il p
pu€'puelsr aql paqleol sralspur pue sAoQtllog (Sl) ;o 'apere; u€r-q+ulro3 aleuro s1r 1o alrds tn '
'ase.rqdered aldurrs e ro;. pue q8ry sda.rols leralas Sntpp q 3uo1 .i
8u1re.u sace; luelcadxa aql se oru ol >laarD qcnru se lsnl e :lr punore sa8ellor aql paJJeMp pue aFe
ara^r +erD slura+ Jo [[nJ s)ooqlxa] Jqquar)s uerlraruv suDlslno aql uo pools'adecspuel aql qtl
atu Surrq plno,lr daqtr uaryo pu€ lpua s,uossal le sselJ dlpnba'rar{lo arLL 'a1dura1 Drroq e ur qpr w

Jo lno uraql 8uq1a8 alqnoq peq 1'rer e;o


qred aqt rc1 @1) soxeryd uo a|'uoq lp sE'srnoqreq o1\{l aql Ic
sarueu qsq8ug aq1 1q8ne1 1;r 1pau,r,r.ed daql'uraq1 qlrrra rcau Ialoq uelpreMpg->laarD asaqo ue se.\r
d.4aod s,urduoda IooqJs aql pear o+ pal4 I JI 'aJuarJs a,r,r aJoJaq 3uo1 alqrsrn'salosar{a oml ara-\t al
ro;8tr.ry1dra,ra pue 'arnl€Jalll roJ Suyqlou parer daql 'sJnoqJer{ IIeUJS lla
'qq13ug lnoqe cueru8erd dlssayqlnr araM daql 1ng ra,ro punoJ sesnor{ a+rq,vr-,trous to uopeJaurolSSe re
plroM ar{+ sdoq ueql asroM ou puE Je$aq ou ara,r,r sdoq (99) e aql'rauJoJ auo olur
aql sa^lasuaql ur pue'palsa88ns peq proJlII,^I ueql ^eI'lsaM-quou 1nq Surqlou :pale-\u
'a)uaps'sanoo'sau1d
.
pr€puets raq81q rct e Jo dlerrurapece ara.tt'sarlrsrairlm pallqewrlm sel'v\ puel$ aql Jo st{lual-au]-\
orq dpo qlrM drluno) e Jo surIlJI^'srals€ur arD qruuuaar8 se lq8q se sautd ueauerralrpay\
dlurepa3'uouwaure8y pa11DI PrlsatrwaldlS araqrur qlr^a para^or ara/vr sIIn{ sll asnPf,aq'
l,rrort quou 1oo1 e ,{po (sauq rvrorrell-uo1g dlpasoddns (og) uala aJeJ se.u dlneaq sll 'lI raqruarual pup
uo tm;) IoorIDS Surpreoq e ur 3tr.ntrlee1 lnoqe pmsqe dru lnqs I uar{,r,r dezvr,e qlearq dru salei
dllueseald Sr4qpuros se,vr arar{l qlIM ulSaq oJ 'loor{rs 'eas Stmrarra lsdqlarue ue rn aler{,rl. {lplq :
uordg prol arp Jo a)uelque rrqoqdorlsnelr arll a>III snua1 raprm Surleog'U rrtes lsrq I ual{
adeosa ol $llez* ro; dpuanbary oB o1 paJroJ sem I qlearq dru Iooi lI 'InJrlneaq dlssaluo$a
'sralsuour lou'sqdurdu (gg) sertt 1r-Bunureq;'anbsarnlcrd'dUard lmL
dq sem 11 'paluneq s€.rr lr JI'puEIsI arp paqf,nol ra^au lanqealpe rarDo ou se1!{ arar{I'lryqneaq se-u
ppq reac 'plro1!t aql uI apnlllos )IproN lsoru aq+ 'alal3 ol derure 3ur-qo1ar1s uea8ay ar{l }o uasaFi
'auaa lsel aql se1v\ lI'apnllos .{q paqruSeur'paur€{ aqt qlnos aql o1 logeladrqlre pa>1ead.{1}uas v
'paleyosr a.raaa. daql ;1 se'aruecgru8rs alqeltmoJreun ol lpools pu€lq aql >looJJ asor{llr uI'sLttPJUflurul
ue uo >Iool-Molaq re; anboec 1u€lsrp e 1o asdurq8 e (99) paxr;1ear8 e'lsam pu€ qlJou arfi ol :tlaslr se alqE.
'qled rrtau e;o dtaaocsrp eq+'a{uqs e;o a8essed aql se adecspuel e Jo Ja+ual aql ut pue snsauuodtr
/suaL[tf Ir
sluala Ileurs lpr{.tr pue 'u€ur aroJaq }sorule Jo puelnelu aql uo sallru xIS lnoQe
-pauaddeg ar{l aroJaq plro^! arp sem lI 'prp dlarer dra.t rarueals ileurs € rn srnor{ Srrqzzep lq31a .{e1 sov
'arfiqJeur
auo lnq 1raraq1e3-utsar € ro '7o33e1a8ng e Surd'r.rec
ueruo.lv\-luesead pazrtoq P Jo'sleo8 palllq-azuorq Jo (sr) Iryarel p L
'a8Bssed Surrn,rolo; aq1 ;o Surpear
(Burzet? ou selvr aJar{l raununs ur) pog Jalurm slr{ pue g1-1 suo4sanb ol sJamsu€ lnod asooq3 '$T--s

.p.ro Surpuodsarroc aql uI q:ea


IIIJ uaql pue uo4sanb qrea ol ramsue lsaq aql asooL{)'uraod.ro a8essed
ragy.aldls pue ,ur,ro; ,lua+uo) rrarp uo suoqsanb pu€ s)rom dreralq ruoq suollJalas Jo slsrsuor uollJas slql- :il
rnoq I-aruII
INOIIf,IIS
NOIIISOdI^IOf, ONV ITUNIYUSII-I HSI-IDNII
instead it was far tougher. The crowning irony was 3. The tone the narrator reflects in his description of
(80) that this obsessive industry, this mole-Hle bhndness to
the island primarily helps to
their nafural environrnent, was what was considered
to be-so tWrlalty English about the system. perhaps to (A) describe the beauty of the sea and the
Greeks, made blas6 by living amongihe most beautiful mountains
(B) verbalize his anticipation of the representation
_ landscapes in the world, there was nothing discordant
(85) in being cooped up in such a termitary; but it drove me of the "Corinthian facade,,
mad with irritation... (C) reinforce the contrast between intellectual
Soon I took to the hills. None of the other masters pursuits and nafural passions
even stirred an inch farther than they needed to, and (D) provide a pedantic explanation for the lack of
the boys were not allowed beyond the cheaaux de towns on the island
frise"
(90) of the high-walled school grounds except on Suniays, (E) analyze his affinity for the physical
and then only for the half-mile along the coast road surroundings
to the village. The hills were always intoxicatingly
clean and light and remote. With no company bui mv 4. The word "obese" (line 23) refers to the
own boredom, I began for the first time in my tife to "
(95) look at nature, and to regret that I knew its linguage (A) proximity of the hotel to the water
as little as I knew Greek. I became aware of ,to."r,-
(B) correlation between the size of the hotel and
birds, flowers, land, in a new way, and the walking, the the island
swimming, the magnificent climate, the absence of all (c) analogous comparison of a taxi to a hotel
(D) intrusion of an ostentatious manifestation of
\afhc, ground or air-for there wasn,t a single car on the modern world on the enticing beauty of
(100) the island, there being no roads outside the iillage, and
aeroplanes passed over not once a month_thes" tt irrg, the island
(E) concentration of buildings on the ,,north-west,,
made me feel healthier than I had ever felt before. I
began to get some sort of harmony between body and section of the island
mind; or so it seemed. It was an illusion.
5. The second paragraph helps to establish the
* literally
a "horse of plank" or a wooden horse narrator's
(A) acceptance of the contrast of civilization and
Excerpt from The Magus by JottnFowles (pp. 52-53).
Copyright
nature on the island
@ 1928, Dell Books.
(B) uneasiness with his first impression of the
island
1. The word "itself" (line a) refers to (C) understanding of the significance of the
(A) "Phraxos" (line 1) facade of the buildings in terms of his future
(B) "landscape" (line 3) experiences on the island
(C) "mainland" (line 2) (D) perspective of the conflict of his inability to
(D) "Peloporuresus" (line 3) escape the trappings of a proper education
(E) "desert" (line Z) (E) critical analysis of the island,s 6eauty

2. The narrator's first impression (lines g-L7) was best 6. Thespeaker establishes the tone of the passage in
emphasized by his use of the fourth paragraph by

(A) pretentioushyperbole (A) describing the feeling of solitude the island


(B) elusive metaphors evoked
(C) metaphysical speculations (B) comparing the isolation of the village with the
(D) whimsicalonomatopoeia absence of wild life
(E) symbolic similes (C) reflecting on the island,s ability to permeate his
understanding of reality
(D) associating his life in school with feelings of
depression
(E) stressing the pedagogy to which he adheres

I52 T TRACKING THE AP ENGI-ISH I.ITERATURE IXAM


t9tl I rsil 1)lllvud

aloq.radlq (s)
uorlrlada; (c)
dra8eun (r)
aqdo.4sode (s)
dpored (v)

aql dq pazrseqdrua sr.ra>1eads aqr


Jo apnlpiJfil ,,
xopered (A)
loqdepur (q) AI'III'II'I (s)
uo4crp (3) AI pue m (a)
m pue II o)
;:j;}tr ($l ru pue'II,I (g)

Id1IJXA Surzvrollo; aql Jo IIE apnlrur a8essed


il pue I (v)
slql Jo auraq+ arp a)rofurar dpq reqr s+uaruala eql .gI puelsr aql uo Stmlreal;o ruruprmuor ar11 .nI
Euelrqegur
s/pu€Isr aq1 yo dqlede aql aq1 .11I
rn apr4rlos
sIF-q aq+ Jo acuenbazuor
s+uapn+s aq+ Jo qsaJa+ur crdodur_aq1 .11
Jo sluaruoru srq dolua;a8uol ou ppo^t. ar{ (g)
$laarD aql pue qs1gug uralsri's
aql uaa^qaq lno >lealq ol lnoq€ r€,vr p se,vr araql (q) Fuo4€Jnpa qsplrg aql;o s8unuoouor{s aql .I

sarldun
1o srmreaur aru+ a,il *J"i?fl:il:i:?o:J:u, ,r, (og aqD ,,ssaupullq a{rl-alour,, Jo asn s,ra{eads aq; .61
drcarp
aurof,aquoos plno^ puel$ aqluo Jaql€am aq+ (g) luau4uroddesrp (S)
IrourJnl Jarrur +uo4uoc uonpprda4 (CI)
plnom s8urlaa; snoruourreq s,.raleads aril (V) uorsryuor ())
sarrdun (7g1 aulf,,uolsnlll ue sewtL, .UI uoqela (g)
uorre4sruJ (V)
luatlgdurr (A)
prua8uoc (q) uer luaruu'rsse srq 0,"];;:;il;?tj:;i;X5ff ,
rrrcqrede (J)
pm8uel (g) pnuanbasuocurdl8urxaldrad (A)
rruorel (y) alenbapell8urpatuorsp (A)
snong.radnsdpnopaz.razro (3)
aqr+eqrqde.r'e.red"",",Jlit;l:#";Ti1"*'",ff ,, )rer{rredlSurdouue (S)
arcorpaudilercqradns (V)
a;q o1 qreordde.rlaql sr soxerqd uo uorleJnpa 1eq1 sagdun ra4eads arll .g
rn paluarro >Ise+ ale sJalseru pue sluaprys aql (A)
a8pa1mou1 Jo uoqecrldde
Jqueruor aqt ueqt ragle; crleur8erd pue dlrnb4ue;o a.raqdsou4" .',n
aq+ o+ uolluaDE JIaq+ sparlp "r"3iHJr?i,i" 1r;
loor{Js aql (cD papnxa
s8urprmor*ns rraql puel$ aql leql Jnapu€r8;o a8eun ue apr,ro.rd (q)
o+ snoJ^llqo ruaas sluapn+s aq+ pu€ sralseru aW
(f,)
Ioor{Js aq+ uI s+uapnls yo d.ro1s1q arfi uo q/ur yo
aql ol lse4uoo pcrxope.red e sapr,rord "r,r"r,o*oJit|l3rT"" ,r,
puel$ ar{+ uo sTaarD aq} Jo apntqle aq+ (A) slua^a sno44da'ns aq+ roJ r"o""f"tfij#Hro (s)
uorssed;o esoql pue palalur Jo slrnsmd
e yo aruolrda aqr q rooqDs
"-Tjt::ffiIfri:* ,o, aq+ uaamleq lDrruuor Sur.dyraprm ue qsrlqelsa (y)

- Jdg3yg Sqzr,rolloJ aqr Jo II€ errofurar ol


sdlaq (gg auq) ,,dre1rru.ra1,, roqdeljru iq1 .ZI
;o n.r, aql uouuraure8y pue er+saurua/13 oi ,".rJrt;ffi .,
Ouestions 16-27. Choose your answers to each of the 18. Lines G-7 suggest that
following questions based on careful reading of the
following poem by Christina Rossetti. (A) the principal narrator is faced with a choice
between the afterlife that true faith offers
Passing away, saith the World, passing away: or the physical corruption that awaits the
Chances, beauty and youth sapped day by day: unbeliever
Thy life never continueth in one stay. (B) although the World has regenerative powers,
Line Is the eye waxen dim, is the dark hair changing to gray the principal narrator of the poem does not
(5) That hath won neither laurel nor bay? (C) paradoxically, life can sometimes emerge from
I shall clothe myself in Spring and bud in May: death
Thou, root stricken, shalt not rebuild thy decay (D) there is a natural cyclical pattem of renewal
On my bosom for aye. that the principal narrator has forsaken
Then I answered: Yea.
(E) the principal narrator is gravely ill and certain
to die before the spring
(10) Passrng away, saith my Soul, passing away:
With its burden of fear and hope, or labor and play; t9. \A/hich of the following lines contains an image NOT
Hearken what the past doth witness and say: echoed closely elsewhere in the poem?
Rust in thy gold, a moth is in thine array,
(A) Line 6
A canker is in thy bud, thy leaf must decay. (B) LrneT
(15) At midnight, at cockcrow, at moming, one certain day
(C) Line 13
Lo the bridegroom shall come and shall not delay:
(D) Line 14
Watch thou and pray.
(E) Line 21
Then I answered: Yea.

Passing away, saith my God, passing away: 20. \A/hich of the following choices best characterizes
(20) Wrnter passeth after the long delay: the speaker's attitude in each of the poem's three
New grapes on the vine, new figs on the tender spray, stanzas, respectively?
Turtle calleth turtle in Heaven's May. (A) Realization of death's inevitabilify; fear of
Tho'I tarry, wait for Me, trust Me, watch and pray. physical decay; passive acceptance of what
Arise, come away, night is past and 1o it is day, cannot be escaped
(25) My love, My sister, My spouse, thou shalt hear Me say. (B) Nostalgia for the earthly world that must be
Then I answered: Yea.
left behind; fear of physical decay; welcome
acceptance of the afterlife
16. How many speakers does the poem directly present? (C) Realization that death will come before one's
ambitions have been achieved; dismay
(A) One
over the visible signs of physical decay;
(B) TWo
(C) supplication for the healing powers of divine
Three
(D) intervention
Four
(E) (D) Sorrow and mild surprise at the arrival of
Five
early death; deepening awareness of death's
certainty; hopefulness for a place in the
17. "Laurel" and"bay" (line 5) are allusions to afterlife
(A) flowers highly prized for their rarity which
(E) Acknowledgment of death's inevitability;
bloom briefly and beautifully and then die understanding of the need to prepare oneself;
(B) spices which add flavor to food and, happiness at the prospect of union with the
metaphorically, to life divine
(C) leaves kaditionally woven into wreaths to
honor poets
(D) traditional symbols for Homer and Ovid
respectively
(E) traditional symbols for true faith and pious
conduct, respectively

I54 I CRACKING THE AP ENGI.ISH LITERATURE IXAM


ggll t IsiI Dtllvud

pueqsnq pa^olaq raq aruoraq


IIr.{ or{^. ueur arp paru dgeryua^a ilwr aqs
salsatldo.rd pos s, rolerreu pdrcurrd
oleq+_ aq; (g)
buruJoru pue lq8ruprur uaamlaq eurq
atuos
lp alrJre grzvr ra8uerls Lr a pue snorralsAu e (c)
papro^e aq louupJ roleJJeu
ledrcurrd
aq+ s+JaJr4 +sJtl ssau[r llpeap
e uaqM rnoq aql (r)
a^rIre rrr^ er aqr* o""T,fflJT;::r"#T;:* (g)
rnoq leq1vl ,{11texa
-
;urmotr{ "q lllvr leql
Jo &uJe}rattm llerus aql
'aruoJ IIrM rnoq leuq,,ro1errn., "1rd."p
fndp.+rd (v)
"r'trl "ro8rn;o ssol pue BuBe (q)
leril +sa88ns 9I pue gI saur.I .gZ

"^",s ";:j::5"Tilyjfi 3]
aJrI AFDrpa (S)
q+eap Jo saruar' ar' uo'errFT::,n;5x;
Jo aseasrp 3uo1 (y)
aql raqury ol VJrqzvr. punore roqdelaru
Ierluar
t1ut":+l sasodord ezuets q)pa qrrqm
.:l?ttr
Aq uorssarHord aql sanurluoJ
o+ u",re+ aq u'r ,]atLno_,, ezvetsorilit"Trofr ,a
ezuels prlrD aql
,,'ta+srs,, pup,/a^oll suJal arfl Burcnpor+urlg
.sezuels G)
asoql dq paJuelpp smar^ allu4s (A)
aql sleada.r pu€ surrrJJeor ezvels prlq+ roqdepru paxnu (q)
. 'sezue+s rarlr€a eq1 yo dra8eu, nq1
aql
.IeMauaJ
e*oq"" Xg (c) luawqwefua (3)
se pazrJalJeJerlJ pea+sur pra88op (g)
sr {e:ap ot poua{{ uaaq per{ rarlrea aqdo4sode (y)
lEqM leql
os Arabeurr leql slseJar lnq,r(raSeurr ,in unif. yo aldurexa ue aprzrord g pve Lsav_I .ZZ
o^{l lsrrJ aq+ Jo rpnul saoqJa ezuu+s prr-q+
ar{I ())
lq pauorrsanb uaql pue aJII Jo sassaJo"rd prnleu pue
aql lualrs aql lq
"rJrAHrH;?;
parue^pe ara1l^ qrnlM saurar{l aql sdolJrrap
paAo4sap r(1enpe.r8 Stnaq sI dpoq,,ro1nrrlr,
(A)
raqlrnJ ezue+s prr-q+ aqr d.ra8eun s1, q8norql
's^rarl Surrafirp
(s) rr"'^ltT, rnrvr pardrur.i:iii.f;,T,:1l", rcl
aJIIra+Je arp dqlromun
,sezue+s rarlrea asoql Bulrcuo"a, lq"i"q1 . Jo +r Japual IIrM
'uorlJe au:os sa{el Jo+eJJeu aq} ssapn,q"rU^
'ezuels lsrrJ ar{l;o d;a8erur arurfurrds
+uaruala alrlrn4sap E surcluoJ
ezue+s p-rroros aql;o .{ra8eur ,{ep Inos s,rolerreu ())
.-_:-Tp"naql raqlaSo;
-btrtppam aseasrp a>IrI_JaJUPJ e ro
salea.t,r ezuelspriq+aqf (V) 'raJue) dq paurnsuoo Buraq,, ipoq s,roleJreu (S)
zsezue$ )L'o)as pue lsrg aql Jo l€q+ ol ezuels prlrfi srporu lq ualea Buraq sr
aW Jo drqsuo11e1a.r "ia;,,,roinrreu (y)
?I:f"i:*! "ql aqr
Alalernrrp lsour sluarualels Buraa,olloy
."r,r"1"nrnq"
.rZ
aq+ leql dldrur o1 papua+ur sr (g1
auq) ..f,e;cie
Jo qrlqr\A aqqi q sr qlow e,, uraod aq1 yo
1xn1.roo arfi uI .IZ
26. Irr context, the word ',spray,,(line
21) most nearly 27.The grammatical subject of the sentence
means that begins
at line 24 is
(A) tree
(B) blanket (A) "Arise,,
(C) a small branch lli ..gsl. is past and to it is day,,
(D) a liquid mist My love, My sister, My rpo.rr",,
lSl "thou,,
(E) a holy spirit (D)
(E)

I56 ; (RACKING THE AP ENGTISH


TITERATURE EXAM
lsrr I IslI l)[)vud

-.raaq Sur.rnoqq8rau aq1 o1 der*e sdrls aq;r'saqtaarq


qsnld pal ur rvtad Jar{ ol alpnord sJtr l slroJsa orIM (09)
orar{ +oo}-xrs aql apqaq aoM 'ace1d luacap e;o adoq
aI$H raq sa^eal qJIqM',rallereql € qll1lt luas os $ ar{s
lnq'Uup€ luas ar{s sr dpo 1o51 'ado1g IIAI
}o asrnoJslp
8urr.r4s-1nos aql Jo pealsur'4,red s,lua8a5 ar{l uI
ileaqlaa.la's e yo dauoq aql o1 Surualsq uaaq arcqo1 (92)
prmoJ sI orl^l preuasnor{ +uearrar arD apqaq aoM'IIe
sapnlrur r{+pqq€S aq} Jo ssar4)rr1s Surlesuaduroc aq}
'pueqsnq rar{ pu€ dpureu sluplras Jaq'papualxa lou
are sassarp ,r,rol pue uopedlsslp aql woqm ol toor
raq raprm asorD ro} Alaleun+roruo awwo (0r)
^eptms
duorr (A) luarudoldua Surraaqc due uror; acuarrlsqe pa;rad
e pue ?lasJar{ dq pea.r uoruras Stmrarra u€'saJlllas
eur8naz (q)
aarql dq ro; pauole 'Iorluo) raq Japun'are >laam aql
eraodoleruouo (3)
Surrnp sassarp rlrol pue uorledrssrq 'alnJ uelre+€qq€S
wduolaur (A)
Jo aJu€ Jasqo lJr4s e dq sr;1as1r slrtor{s ,{cuapual sr-ql (gt)
aloqradlq (y)
qJlr{zvr uI lurot aqt pue iuerrrom snor8qar e derur uznto
Jo rar{ ur $ aqs 'ppor* ar{+ Jo sJauu€ur pu€ dlarcos aq1 o1
aldurexa ue sr g auII ul,teln+Il, pror! aril Jo asn aqJ '09 asrale 1ou q8noql 'pauoquaur aq lsnru a;rrvr s,doqsrq
aql Jo ral]€r€r{J aql uI dlrreqncad pa>Ireur Jaqlo auo
'qe+lP re^a (0€)
(rg-gg saqD
seq,, G) u€r asnoq u^{.o srq r{Jlqlvr acead o1 goeo.rdde lsar€au ar{1.
,,srapuaJJo qJns roJ sn8ry;o sada aq1
(LV-qV sardlJ) sacnpo.rd uolssnuqns lprp areme sr pue 1uor1erryl1snl
rW asrnoJsrp Sutr.rqs-1nos aql,, (q) -;1as qdwape uala r{1arer aq'paept4 lwlr{ ruorJ passed
,,ado1g Jo
GZ-1Z saurl) ,,aread
3uo1 seq lasturq Surpuayap;o adoq ilV'lqBF ralau sI
o1 qceo.rdde lsaJeau aql sacnpord uorssnrrqns,, (3) aq atrm srq;o sada aq+ uI ter{+ uraas plno^{ 1r'pllozvr eW (SZ)

(ff-gf sauq) ,,>luer rar{ ;o sar(a arD ut JaaJeJ srq uaaq s€r{ se Iryssarcng'rrlodsap
$ aqs pu€qsnq rood raq ol lnq'11e o1 arrqeluor{lne
;o sa8aprurd IInJ aql aurnsse ol rvroI szltorDl (g)
(8 ar4D dpnlgqeq sr dpel snIJ'alpnord srtr tr os +oN
'alnJ
,,uorururop aruor{ qJns qlIM pal}s4€s lou,, (v)
s/ueuroM e Jo s+null ai{+ aq pFoqs +€tlx\ sMouT aqs $q (02)
aserqd agl ur dlpalurod lsour 11r a,rrnbre ol uaAIJ+s d1p;ssacrnstm lou ser{ pue'rarvrod
u.I!^.or{s sr ralJer€qf, uelr€+IJoqlns s,alpnord sIlN '62 sanlel ar{s ssallqnop ldreqs $Iool rar{ rou'pnol ralau sr
aJrol raq laJuarpaqo 1o uralled e sI ar{s plrom aq+ aroJaq
(OhSg saurl) ,,depuns aq+ uo luaurdoldura lpueqsnq rar{ saru€r{s ra^au ar{S 'luarlJauaq pup
Surlaaqr due uorl aJuaw+sqe 1ta1.rad e,, (A) sr'seq aqs dezvrs;r dezvrs s,d1+uer5 srry lng 'ace1d^s€a
pue (g;)
auol Surruoraq uI prnru u-l!^.o rar{ ssardxa pue'>IueJ rar{
OZ-SZ sauq),,1q8r'r ra,tau
sr aq aJrzlt stq;o sada arp uI ruaas plnom lL, (61) ;o sa8aprurd ilry aql alunsse ol rvroq smouT'pealsurnld
(6I_8I SAqD lp auror{ dddeqlaq ul'ayr.tr s,uoJeapr{JJe aqJ
'pa>lcaduaq
,,1r arrnbce 01 ualr4s d11n;ssarcnsun
sr doqsrq aq+'+JBJ u1 'pqrrrds s8tqp woq rnelsqe (0/)
+ou s€r{ pue'rarvlod sanl€A aqs ssallqnop,, (J)
(ff-Of sauq) ,,paloaduaq sr doqstq aq1,, (g) uala lou IIII\ pue'sluarua,totu sn{ II€ JaAo raanod;aq
(Z-t saqD ,,alpnord sr;41 lsure8e sar{f,laqs pu€'uonnurop aruoq qDns rDI1v\ paqsr}€s lou
prolrt e aqlearq ol uoqualu dru 1ou q lL, (V) sr arpnord sJI tr +ng d13r41gm, pr( 'dprelrmlol lou Jr 'rar{
ol pauopueqe aneq lq8gr alpnord rC f,4saurop s8qqI
2duo,n;o asn s,Jo+€JJeu aql Jo '1e sr-{l sr roN'uorr Jo poJ e q}IM salnr pue'prol relnlp (9/
alduexa ue aql Jo i{Jr,qM '82
sr suo4dr.rcsap SurzvrolloJ rar{ JaAo aurardns salnJ ar{s Drlsaruop sJailEru rq leql sI aurT
qlnr+ aqJ 'ssaurddeq s,pueqsm{ raq ol r{Jnur spp€ aqs
'qleqq€S aq+ Jo uoqelrasap e a'rr8ro; san+Jr^ raq il€ qll^{ leql >FryD louueJ I II-qs rnq'alpnord
arpnord srIN IIII!{ durouoca ro rnapuer8 roJ uala lou lnq srtr41 lsure8e pJoM p aqlpaJq ol uollualur du 1ou sr 11

'qr11d q8ry e 1e 1da1 sdezwp aJ€ slerour ar4;r parnoord


'ado11or1
aq o1 dlpreq ale sa8e.u, ,vrol uo laal xls 'ro;'paloopa,lo (99 )
aq.{eur >laam ar{+ w ssauua>lurup puorserro 'srapuatJo duoqluy f,qsnmol n$a4Jfig Ia^ou aql tuorl ldrarxa
r{rns roJ sn8ryyo sada aql seq alpnord srtr\l'asn srq ol up sr uorpalas aql 'sJamsue.rnod asooqJ nod aro;aq
palerrdordde leas >peq aq1 q 3q1py;o pealsur'doqs dlryareo a8essed 3um,r,o11o; aql pea1 T7-8-Z=u6trs5fr-O
the context of the passage, the phrase "if not vol- 36. In context, the adjective "recreant" Qine t14) is best
3L. La
interpreted as meaning
untarily, yet willingly" (line 7) is used to show Dr'
Proudie's attitude toward (A) unfaithful and disloYal
(A) the duties that the clergy are expected to
(B) engaging in a Pastime
assume
(C) refreshing
(B) entering the institution of marriage (D) craven and cowardlY
(C) strictSabbatarianism (E) depraved
(D) granting his wife some Power
(E) the hiting of domestic helP 37. What is the effect of the repetition of the phrase
"Woe betide..." in the final paragraph?
32. The description of Mrs Grantly serves to (A) It retards the tempo of the prose'
(A) provide another example of the power of the (B) It satirizes the fate of the servants'
aristocracY
(C) It highlights the drama of the situation'
(B) prove that Mrs Grant$ henpecks.her husband (D) It chinges the point of view of the narrator'
(C) imply specific faults of Mrs Proudie igj It emphasizes ihe moral consequences of the
Proudie action.
inl t"ggltf a rivalry between her and Mrs not
iEj utJ-".t *hy women should be seen and
heard 38. In context, the word "chatacter" (line 31) is best
interpreted as meaning
33. The narrator's attitude toward Mrs Proudie can best (A) dubious Personage
be described as one of (B) reference
(A) pity (C) antagonist
(B) objectivitY (D) conscience
(C) emotionaljudgment (E) footman
(D) sardoniccondemnation
(E) jaded disgust 39. Which of the following best describes the effect of
the last ParagraPh?
34. \ /hich of the following best describes Dr Proudie's (A) It suggests a cause of Mrs Proudie's moral
relationshiP to his wife? transformation.
(B) It introduces Mr Slope as an observer of Mrs
(A) MorallY devoted
Proudie's actions.
(B) ComPletelY servile
(C) AwkwardlY tender (C) It illustrates how Mrs Proudie's religious
beliefs reflect her character'
(D) ThoroughlYuxorious
(D) It counters speculations about Mrs Proudie's
(E) BitterlYtYrannical
character.
(E) It shows how hard it is to hire household
35. The author atfributes Dr Proudie's attitude and servants.
behavior most clearlY to
(A) ambition 40. The style of the passage as a whole can best be
(B) pride described as
(c) pacifism
(D) spirituality
(A) humorless and Pedantic
(E) feudalism
(B) effusive and subjective
(C) descriPtive and metaPhorical
(D) terse and ePigrammatic
(E) wittY and analYtical

I58 I CRACKING THE AP ENGLISH TITERATURE EXAM


6il1 I IS]I lllllvud

'1no 1da,ra.s'a1nor1ar.raq ur.raqdosopud rIJuaJ{ € prrv


p>1ood rar{ w uaru poo8 uarrala q+rzrt';aq1otr41 sy Gr)
laTJop aq1 Surdpn+s 'paleSora.radns 'a8pnf aq1
'loll€q Surgurs /1eulJ aql Sur.rnp a8ed 1se1 aq;
'uala lno auror Surzreq'8ueq e qlIM aJrello1 +nr{s ar{S
'arualuas Surctmouord raqlontr a>lII paooJ ralaN

Io4uo) ou per{ sa^oQ @?)


s€q aq qrlq1!( JaAo saJu€+slunJrlD Jo luIlJI^ e ([)
e (C)
,,>tuEd arD >IIe^.,, o+ apelu aq III.\,'. or{1!^' ueur iraru€J q+rm paqlnoru-ualloMs 'lua4ed srq ^tou puv
+srreuluras e (J) '''salol'sllqeq puosrad'acue.readde'aJr+)eJd
Trads-du € (g) 'uorssayord 'tlulq dq'pauroop 'pawoop 'pauoop sem aH
ssauIIT leurura+ e ruory Surragns ueru e (y) :1\^ar^ auqueru€pe Jar{ o+'alrJop'sJaquraur-1talJ
ualala aq+ pal'sluaurded dsea srq ile {oo+ rar{ro4 (gtl
aq o1 paqdrur
q '€t 'an8uol s,urn)rl srq q8norql Ilrrp aql-islql {r€IAtr
d1pa4p +soru $ +squap aq+'€zuels puo)as aql
arl sunJ's1oo1 ranps srt.l JaAo leq fuarrls 'ra11
-lllJp Surquror'raq 1e loodplqrvr du11 aql lsed sarelS
sleads lsquap '8rmr1stm 14s lluaqed € s+rrup€ aH
drun€ qDIq& q auol larnb d.ra.t aqt 1sa33ns (s) 'prara51 Jrls€ru sr-q ruoq 1se1 1e dezu.e 3u1p1 (gg)
"ql uorsrJap rar{ saqJear 'sllg r.uoor uo4daJar arD s€ aJueIIIPp rraql aro)s
rar{lour aq+ qJrqm q+mt paads aq+ azrseqdrua (c)
IIaM reaq lou saop raq+oru aql +€t{+ 1!(oqs (r) sarun{J leJrsnur lt{.aN
I€rJl ar{+ se Surpear o+ uoqua+le
q)nur se sdedlaqloru aql +eql peJ arp aJroJular (s) 'I{JUIIJ arD le pauuIJS
dlrapla se rarpour aq+ azrra+)em{f, (v) qlaa+ .ra1se1d Jo saluorleq IInJ araqa
ol salras (S aqD aserqd aql 'ZV qe1 alftl ar{+ prmor puB punor'fq s,11op raq qlIM (92)
,prea:q1-Ile:q.,,
as.rnu Surdeld 1ape1s e a41 'paddq-asrnd
'lsruorldarar lrr8 ualrs srq paser{J aq a,t.oq :saslU
aql a^oq€ Jlasraq sJaplsuoJ oq auo (g) duourrlsal aql se aulrq 3rn1se1 1de.ra.e ldarvrg
^rel ^
dpq8U oo1 rornf e sa8prrq a>l{-aJII sn{ il€'ssalule4 dllaruro4
se sarlrlrqrsuodsallaq 3ur1e1 qlIM pauraruoetm (6)
'trnq sasdurq8 aqs 'qldap raq puodaq ra.r.ag (p7)
dlpn8 sdezvrle +sorule
'a{o4s lsearq dsea ue qlIM II arunlo1 q8norql 8.4pIIC
are IErq o1 lq8norq asoql leql sa^allaq oqzu' auo (3)
uollenlls.raq dq raq uodn 'sasllra^p€ lsrluap sHl'II€ ]o +sroM
pasodun saqnp arfi Jo ar€Metm dlapldruoc (g) 'aseald '1no 1r 1rd5-JaqJeq 'ra>lzvreq 'ra>1ceq 'sraqd ;6
paraplsuoJ pue paJa^oJun rapIaI^{
aq S+JEJ lu€^alar aql il€ 1er1tr Srnaas ur palsaralur (y) 'raplart'cruerpau aql sa8pal.r,toupe aqs'Srnzaa.rg (91)
aq ol raq smor{s uraod aql rn pa}elar 'saal8ap alseqD aql'ldlrJs ur1e1 a1qr8al1
IqIJI aq+ pJ€Mol apnlpl€ s,rar{lolu aq+'ll€ra^O
'I7 'a1e1d sserq alqepadsar aq1 's8qpd uorl ar{J
puodaq are slsquap lsuenrsdqd JJI+s JO
arnl e ruoq papuaJsap lssaupaprnurlet rar{ JO
r""iTf;#j >Irar^( aql pure /arrpllo^'raq ol Irre) (01)
aw .,, sppruratr/{ uro{,,sro})ocl;o aurl 3uo1y,,
ilJo u4r{ qsnd pzvr aq5 'peu.ra8ur.; rar{ qrIM
'a4pllo1 a>lrT'alqesuadqpnt umr smouT aqs lcads-dg e lcrg'a8ed e surnl'surra.o.r1 raqlot'{
:aJ€J pue arnJ sIH rOJ Saseaslp slua^ul 'q8noo snolrau e srapuar'>1ue1d aql slunotr I
'luaruPodaq (99) >Irap aq+ ssor)€ pal $ ls4uap a^r+mJ aIDII aqJ
/+r
ry auar8dll uI sl)nrlsur'acrlre,rd leraua8 e se11 '1tau paleu srq Surreq lnor{}Im dn sa1oq3 (g)
'urtoul 1a.rt slal€lopr anr+ aql se'oq A 8uq1eo sn{ sI JelIoJ asoqm lsrJerfluras e sY au17
dUeC Jo uolsral umo rar{ Sursodurl
"rD llgn8 sea,r lqluap ar{l'prom B preaq-Jl€r{ aqs aroJag

,,'arar{l lq8p'a1q rnod 1no 1rdg,, aqs'dervrduy'sournlol aalp q a4u41o1lo afi7y


zra.arl>I

'des uaq+'dernte urrq mry o1 'tq urlq padde'4s pW bS) q8no.rq+ pB o1 pa8eueru drnp drn[ ro1 pa>1o1d
/surJe ar{+ Surddr'rD "raq1otr41
'uM.op padoorvrs ppr{ aqs asn€Jaq
'rreqr dsearm s1q o1 Sur8urlo'1oo; s,a^ol lV '.e\olloJ leg1 suo4sanb aql 01 sJaMsue asooqJ uaql
>lJ€q >lool rala aqs prp 'srsatuall 's4 I acrN pue'd1p1arer olaq uraod aql pea5'EE=ftTiroJJJa-7fO
^
44. Which of the following best conveys the meaning in 48. The phrase "tasting brine" (line 22) indicates the
context of "Freezing, she acknowledges the mechanic, dentist's
welder, wielder/ Of pliers, hacker, hawker, barber"
(lines 15-17)?
(A) desire for the trial to be over quickly
(B) anger at the falsehoods offered as testimony
(A) The mother thinks of other tradesmen she against him
dislikes as much as she does dentists. (c) shame at the revelations of his unprofessional
(B) The mother thinks of professions similar to behaviour
dentistry. (D) fear of being imprisoned for his acts
(C) The mother thinks of the diverse and (E) sense of the growing likelihood of a guilty
distasteful aspects of the dentist's profession. verdict
(D) The mother thinks of trades which, like
dentistry, she recognizes as necessary
49. Which word is used as a metaphor for reading?
although disagreeable.
(E) The mother thinks of the control typically male (A) Dreaming
professions exert upon her and how in this (B) Walking
instance the tables are turned. (c) Conversing
(D) Swimming
(E) Flying
45 The phrase, "this dentist advertises" (line 18) princi-
pally suggests that
50. Which stanza suggests that the mother's treatment
(A) the dentist is unscrupulous
of the dentist could be seen as "poetic justice?"
(B) the dentist is not professionally qualified
(c) the dentist's lack of skill causes him to (A) 4
constantly seek new clientele (B) s
(D) the dentist is a newcomer to the area (c) 6
(E) the dentist offends the mother's sense of (D) 7
propriety (E) 8

46. The poem states or implies which of the following? 51. "Nice Mrs. Nemesis" (line 47) is an example of
I. To a large degree the mother finds the dentist (A) understatement
guilty because he is a dentist. (B) hyperbole
II. The jury finds the dentist guilty. (C) irony
m. The dentist should be found innocent. (D) personification
(E) onomatopoeia
(A) I only
(B) II only
(C) III only
(D) I and II only
(E) I,II, and III

47. In the fourth stanza, the dentist is portrayed as


(A) comicallylecherous
(B) brutally vicious
(C) calculatinglycriminal
(D) timidly amorous
(E) angrily frustrated

I6(l f CRACKING THE AP ENGIISH TITERATURT TXAM


l9ll I Islr l)rl)vud

'OS OCI NO OD ION OCI


.NOTtf,iIS ,CIg'I'IVf,OI
OI CI'IOI !ruV NOA'II,INN II NOII)gS
SIHI NO XTIOM UNOA ))!IHJ AYI^iI NOA SI SNUI iruOdgg HSINIC NOA CI
I NOIIf,IIS {O CINII
dOIS

(Sl aqD ,,alncBar rat{,/ €)


(Sl au1D ,,raqdoso11qd qJuar{,, (c)
(Sf aq) ,,uaur poo8 ualala,, (J)
(St aqD /,raglotrAl/, (s)
(m aqD ,,a8pn[ aqg,, (v)
patqns
EI s€ sa{el (Sl aql) ,,lno ldaals,, 'dlprrleururer5 's9
a)ual€Arqru€ pasnurv (a)
aruaraJlpur pIoJ (c)
arns€aldslp [nJuroJS (J) dpo111puel (S)
aluEr{s snornrv (s) dponIpuel (CD

JeBuE pal€4sruC (v) druo n (r)


druo rt (s)
ZaJr^ras ,(m[ s,raqloru aql pJ€Mol apruFl€ s,pod aql dpot (v)
saqrrrsap lsaq sarroqc 3u1mo11o1 aql Jo auo qrTqM 'gg
'urnroJap lelcos rol paau aq+ ur
luaued e Jo luau4earlsrlu sH (:I) JaIIaq raq sareqs poD l€r{1 sa^arlaq laq+oru ar{I 'il
rou€aruap sr-q (A) 's,aJr€llo^ to asoql ol JEIIurs
lu€lsrss€ sr-q qll/v\ rreJJe sr-q ()) aJ€ s1v\ar^ s/poD lpql sa^arlaq Jaqloru ar{J 'II
uorssayo.rd sTq (S) 'palecgsr,qdostm
slalaq sno€qar sfq (V) rnq plaq dldaap sI poD Jo uorslA s,rarilour ar{I 'I
IcIiIf, XA suos€al Surmogo; 23u1ru,ogoy
aqr Jo IIe roJ lslluap aq1 ;o saaorddesrp .raqlour aqJ 'tg aql Jo r{Jrqm qsaSSns e Jels pug s,ruaod aq1
ENCLISH LITERATURE AND COMPOSITION

SECTIONII
Total Time-2 hours

Question 1

(Suggested time---4O minutes. This question


counts as one-third of the total essay score.)
The passage that follows is excerpted from Don Delillo's
novel Libra, (19gg), a fictional treatment of the young
Harvey oswald' who as an adult wbuld assassinate President Lee
John F. Kennedy. Read the passage carefully. Then write
a well-organized essay concerning the methods
by which the author nas portrayed the subject and the
portrait itself' Be sure to consider such literary elements as substance of the
dictiory imagery, and point of view.
He returned to the seventh grade until classes
bundles on the wicker seats. He jumped the turnstiles
ended. In summer dusk the giris lingered near the
once. He rode between cars, grfppina the heavy
benches on Bronx park South. chain.
Jewisi girls, Italian girls (45) He felt the friction of the ridJin his t-eeth. They
Line tn tight skirts, girls with ankle bracelei-s, their voicJs went
so fast sometimes. He liked the feeling they were
(5) murmurous with the sound of boys, names, with
song on the edge. How do we know the mJtorman,s not
lyrics, little remarkshe didn,t always understand. ThJy
insane? It gave him a funny thrill. The wheels
talked to him when he walked by making him smile in touched
off showers of blue-white iparks, tremendous hissing
his secret way.
(50) bwsts, on the edge of no-cor.rt ot. people crowded
Oh a woman with beer on her breath, on the bus in.
e-very shape face in the book of faces. They pushed
(10) corninghome from the beach. He feels the tired
saltv through the doors, they-hung from the por"ulul., straps.
sting in his eyes of a day in the sun and water.
He was riding just to ride. The noise had a power
"The trouble leaving you with my siste1,,, and
,,she a human force. The dark had a power. He stood
Marguerite said, had too many children of her at the
(55) front of the first car, hands flat against the glass.
own. Plus the normal disputes of family. That meant I The
view down the tracks was a form of power. It was a
(15) had to employ Mrs. Roach, on pauline Street,
when secret ard a power. The beams picked out secret
you were two. But I came home one dav and saw things.
The noise was pitched to a furyhe located in
she whipped you, raising welts on yor, l"gr, and we the mini,
a satisfying wave of rage and pain.
moved to Sherwood Forest Drive.,,
Heat entered the flat through the walls and
(20) wtndows, seeped down from ih" tu, roof. Men
on
Sundays carried pastry in white boxes. An Italian was
inurdered in a candy store, shot five times, his
brains dashing the wall near the comic_book rack.
Kids trooped to the store from all around to see the
(ZS) of grayish spafter. His mother sold stockings in
lr1cey
Manhattan.
A woman on the street, completely ordina ry, maybe
fifty years old, wearing glasses ind a"dark dress,
handed him a leaflet at the foot of the El steps. Save the
(30,) Rosenbergs, it said. He tried to give it back
if,lrrt ing
he would have to pay for it, but she,d already rurnJa
away. He walked home, hearing a lazy radiovoice
doing a ballgame. plenty of room, folks. Come on out
for the rest of this game and all of the second. It
(35) was Sunday, Mother,s Day, and he folded the leaflet
neatly and put it in his pocket to save for later.
There is a world inside the world.
He rode the subway up to Inwood, out to
Shgepshead Bay. There were serious men down there,
(40) rocktng-inthe copper light. He saw, beggars, men
who talked to God, men who lived on ilie trains, dav
and night, bruised, with matted hair, asleep in patient

I62 N CRACKING THE AP ENGTISH TITERATURT TXAM


t9lg I rstl l)tllvud

uerrrauy druapery aqa dq ffi02-l66lO lq8pddoS ,", ,"r-Uri|;:


1o
sur11o3.rad.re11 l:lrzu. luaruaSuer.re
xq pasf p"-asar sfq8rrlly
pal'lq9g6l O lq8pddo3 '996I ,MoU ,"d"n11 lq paqsrlqndl ;r{b;;
ry ruy *oij

qle14 eu.1dg-
'suoolleq a{rl asrr sla.\,l.o^ realJ aql
t4 nod ,*ou puv.slers rnp.r, #:ilf.tHflH#n
arenbs ,vropurM ar{J .s,leJ e se u€alJ suado qlnout ;rr.ro1 (St)

relorr pue rneaq-.noa ,r", J#:il#fiT#?j:H


rea dur ur salour eas JeJ V
:uals{ ol a)e,vr I .sasor 1wd 1ep aq1 Buoure sJa>p{C
q+earq-q+otu rnod lq8ru
a{elg ureqlrM_ lly @f )
lsearq s,raqloru Au uodn >llns oI
Mors uMo srr rraual o*o,,,,1X?,'o5#ffi;L:ijil,
,,0,"1?ll,Ttol'Jll j',","#"ffi x,T,",g Jaqloru rnod aroru ou ur,J ",*
'spueq s,raqleJ dur ur 3w188n4g (S) 'sllelvl se dpnrntq punor puets a14 dlaps
rno sMopeqs
'pnolJ
au!7 .anle+s ssaupa>leu rnod ,runasnur
.lelrrre rnod Bur,,(yru8eru,oqca fiyerp e q (S)
e l.r[ pn{ puarJ e a>lrf uuraN sarrol ,n[ iu17
_'pnol Surdrd,paleu,ssaldpll
,1deay
_ J plrom sno.ra8uep aril oluJ rrcppqrnodpue,.":,HiH;t:;T;#H,,"j,"i1;l,.lh*
11dau raqle; dru,paueol8 raqloru dyq
't{4EM plo8 1ey e a1q Buro8 nol ps aaol
Morros lueJul
Suog Stmrrop

drue;ur premo+ sapn]Fl€ rrar{+.lea^ar o1 .,ira8eurr asn sraleads


aql urtorl az[leue dessa pazrue8ro-qazvr e ur,uagl .a4e1g UeIIIIM
pue qteld ernldg dq suraod Buuaoloy aqt pear lpyare3

('arors Aessa Ielol alD Jo prrql-auo se slunoJ uo4sanb srqJ .sa+nunu


0'-€u4+ palsa88ng)
7 uoqsan[
Question 3

(Suggested time--40 minutes. This question counts as one-third of the total essay score.)

"When a true genius apPears in the world, you may know him by this sign, that the dunces are all in confederacy
against him."

Swift
-]onathan
"Thoughts on Various Subjects, Moral and Diverting,,

In some works of literature, the main character often finds himself or herself in conflict with the social or moral values
of his environment. Choose one novel or play of literary merit in which the character is at odds with the people around
him or her, or with society at large. Write an essay in which you explain how these conflicts are essential to the overall
meaning of the work.

You may select a work from the list below, or you may choose to write upon another work of comparable literary
merit.

The Awakening Man's Fate


As I Lay Dying Mnrat/Sade
Catch-22 Medea
Crime and Punishment Miss Lonelyhearts
The Duches:s of Malfi Moby-Dick
A Fan's Notes Native Son
Hqmlet Nausea
Heart of Darkness OId Goriot
Hunger One FIew Over the Cuckoo's Nest
I Knorn Why the Caged Bird Sings The Scarlet Letter
The ldiot The Turn of the Screra
The Iliad Under the Volcnno
Inaisible Man Waitingfor Godot
King Lear Wuthering Heights
Long Day's Journey Into Night

STOP
END OF SECTION II
IF YOU FINISH BEFORE TIME IS CALLED, YOU MAY CHECK YOUR WORK ON THIS SECTION.

I64 I CRA(KING THT AP ENGLISH TITERATURT EXAM


QUEST|0NS r_15
by British aurhor John Fowles, is a
il?.t-ffi1f,:TTrlT"i":1"::,'"r" 1t^T!:!:q,:,.1 :ou,"r serecrion
lli:?Y*'h:#flfi:"::i:":f t':*tm;rililq;'q;#rffiff ?;ffi:yi.-';#ffil".'*";
:::li5ffi:f
fi
those elements on iftie:,:*::y:,1,.,.:l17erements,u,,"in"*"*i'rilH'.lTffiilXHH.Til
selection r,,;h;l'..

T^",::::1S: ""lr"lor is imporranr in rhis piece, as


T
tlit it every piece. But after you have
.rn
ffi'"T"Xl:l1p"ll:,,:.f:,,""*y",i,no,1J;;;br.i;*",.",r'f,fJt"#.."?,i",::ffiJ"?Jil:
situation is reflected in the setting.
1' A You may have read answer choice (A), which is
a straightforward answer, and thought,
"oh' that's the answe1," and then puzzledou"r *,.-oii?r
choices wondering if you,d missed
something' Perhaps (B) and (C) seemed
to ue possiut. unr*.r, also. Most
of the time a
Pronoun refers to the last noun that has been u"t there are-e"..d; to this rule.
In this case' the narrator is telling us ""*J,
that phruxos, *hich is in the centei
(B), is as memorable as the landslc"p..jr. oi th" tu.ar.up"
I,vhen you tootea at choice (c)
aga*; you wourd
ror rocatioir ra** p,limagery crJrce ror courd not
ff;:;::lfrT;"T:S:ilS;J#J:a precede the pronoun' not the other
around. Your only logical choice way
2' E Remembtt tlil^I|* "L"#r?fust
u-t given choices that contain two words,
I.ot
rectly answer the question' both words must cor-
If oie of the wordr ir then
be eliminated' Thd question also the entire choice must
"of".."rate,
tests your knowleage or hterary terminology. Choice (A)
could have been eliminated immediat"iy
becaur.iir.""**ror states that it #as ,,simply
and effortlessly beautiful'" Thatalone
would have eliminated choice (A) because ,,preten-
tious" indicares i! was showy and not
simple, rh. a.r.ription is arso no1
perbole. Choice (B) could have been . f;;; of a hy_
a close choice b";;;1
ptrur" "amethyst;t."t"g;;;;^il. udlectl'e;urned;rtt:#;!:::l1ff:?5i:l;rT$;
However' the term "elusii'e" eliminates
the answer ul.urr" ,,elusive,, indicates
intangible and mysterious' The concrete that it is
comparisons that are used in lines
help to eliminate this answer. choice (c) 5-10 would also
indicate, tr,uiir,u comparisons that
farfetched and do not have any concrete are made are
references. This is not the case.
that there is no depth to the impression Choice (D) indicates
thelandr*p";;; made on the narrator. This
exact opposite of what has occurred. is the
(E) is tn" .orruit ur,r...r. The
these lines are predominately similes, comparisons used in
and when you .orrtirr.re reading,
use of venus and the use of ihe it is evident that the
whale ure symboii.. v.""r, the brightest
is used in contrast with a black planet in the sky,
whale **.ni.,g ,.u.hro, venus
and romdntic notions, whereas the 1 is symbolic of passion
whale ir u ry*u'oio?.urti,ry and pragmatic
ideals.
3' c Make sure you understand what this
question is asking. To answer it, you
about what the narrator's tone is need to first think
as he describes the isiand H.:,
Now' what is the effect of this awe? Choice i."ii;;;y its beauty.
*.-r
tal p..tty redundant'and does not really
address how the tone advances
somethint.il.;;ii;;;"".. his anriciparion of the
"facade?" This choice is only partially one
correcf the narrator is expressing something
the setting' However, about
(D); this does not really suit the
.-pil;;;
he is noioffering a logical
of the lack of towns on the island
toneif
y; ryu!,*ryi;-. thing with (E)-the narrator
is riot offering u objective analysis. so what aou, i,i,
:?Jd.,
beauty do? rthighlights ihe contrast'befween
u*ed description of the island,s
nature and the intellect, choice (C).

I66 T CRACKING THE AP ENGTISH TITERATURE


IXAM
t9a - sN0tlvNvtdxl 0Nv sulllsNv :l IslI l)ll)vud

-rldarrns ou are araql asneJaq (g) aoroqc uasoqJ aAEq lou ppot{s no1 'uoqsanb srql ol ra.MS
-up parro) aql sapr^ord prguor sTqyaJIM slq dq paraprnru aq ol dpo'd1a;es aruoq paurnlar
uouurarueSy'JapJnu aql JoJ uouuraure8y uo a8uarrar panrol 'dorl;o ualaH snorueJrn
aql ol Jalsls oqe 'e4sauura1d13'aprvr s41 :alq8nep sr-q 1pl ol uoISIJap sn{ pazITeuoI}BJ aH
.d1aps aruoq rirn+ar
lq31u aq leqt rapro rn spo8 aql o1 erua8rqdl ralq8nep sl{ parlJlrres aq
,arn4redap sr-q ol JorrcJ 'dor1 o1 snassdp6 paruedurocre oqm JoIJJErvt
IaaJD e'uouurarue8y
Jo aJIM aql s€rvr e4sauura1d13
'nod Jo pajse Suraq sr lur{rvr puelsrapun lou lq8rur nod
uaql dSoloq/ru pue dpa8e4laarD ur areld rrarfi rraoDl lou op nod I'suolsnile are uou
-wauie8y pue e4sauuafi3 yo sarueu aql leql IJEJ ar{l q}rrvr nod saprrrord uoqsanb sql V 'L

'ssa.rdxa

ol Surtrl sl roleJreu aql leqm;o Surpuelsraprm qldap-rn arotu e sapltord 11 'uousanb sryl
roJ ra,r,rsup lsaq aql sr (3) acroq3 'qder8ered sTq+ ul pasn Suraq lou $ auol dpeloqx e leqt
'uotssardap
luaprla s111 'arualuas lsel aql ur sralsuoru pue sqdudu Jo uoquaru aql qllM
saleJrprn spJoM asoql Jo Jarl+laN t;arraa,, pue ,,'aJueJqru3ts,, spJoM aql Jo asn sr-q dq luapl^a
sr TIJII,vr'qder8ered sr-qt q apnlple s,JoleJJeu ar{+ }o alnpld alurnrre ue apnord }ou saop
(6) atroq3 'salels rolerreu aql suosrredruo) aql Jo auo o1 dluo srayar (g) a)rot{) 'puel$ aq}
Jo apnirTos aql;o uoudursap aql ol sraJar dpo
(V) a)Iorlr aser srql u1 'paprlord seq roql
-ne aql 1eq1 dra8erur aql pue a)roqJ pJorvt aql raprsuoJ lsnur nod'auo1 qqlqelsa ol rapro
uI 'uorsrJap B a)eru nod aro;aq qder8ered arqua aql le {ool e ale} lsnur nod'ue8e arug )'9
']Jarro) sr (3) 'aro;a,raq1 'a8essed
aql rr ralel lraga lear8 ol uodn papuedxa q leql uosrreduor e dn sps ll 1lur0 ueql aroru
saop qder8ered qq1 1nq'ftneaq s,puel$ aql;o srsdpue Ierqlrr e sraJJo qder8e;ed puo)as
aqI 'uoqsanb aq1 srar*rue ro luaualels aq1 salaldruo) lsaq leql a)Ioq) arll lJalas lsnur nod
lerll raqruaruay 'a8essed aq11o Surpuelsraprm rnod;o anrl aq deur s1q1 q8noql uala '(q)
arror{) r{sllqetsa o1 dlaq +ou saop qder8ered puoras aql 'uolssardq qq;o uoudursap alern)
-Jp ue lou sr (g) aJIOtp uI ,,Ssaulseaun,/ pJolvr aqJ ,,'saJosa f,a,, aqol s8uppnq uJaporu aql
sJaprsuoJ pue dlneaq aldurrs Jo sruJal rn lI saas aq pupl$ aq] uo saAIrrE aq uarl A'a8essed
sr-ql Jo anr1 lou sr ,,alue1dalJs,, pJolvr aqJ 'aleJnJJ€ lou $ (y) actoq3 ,,'uolsnlll,, ue osle ale
seq aq duouuuq;o s8urlaal aql leql srneal all 'asrn8srp E seq ter{t 3tm4 dpo ar{+ lou sl sHi
'aloqm € s€
leql surual ralel aH ,,'ape)eJ,, e seq Suppnq aql 1eq1 sazruSoJar lolerr€u aqf,
uorlJalas aq1 ol qde,r8ered puocas aql Jo aJuel.rodun aql uo palJar o1 nod slse uoqsanb srql J'9
'(q) acroqr le pallrre a^eq ppotls nod'gg4
q8norql'uoqJalas sH+ ul sprom aql Io [e;o Stmreaur aql l'rouT ]ou p1p nod;r uarrg 'aders
-puEI aql uo uolsnrlrn ue pue rusqenraunuoJ urapou Jo +lnsar aql $ Ialoq s1q1 'ace1d Jo lno
dpuapr.ra q l€ql pa[qo a8rel dral e salldun tpsaqo,, proa aqJ'(d sl aJlot{f, Stmneurar aq;
.uoueJol IDI,r,r op 01 seq
lerp qder8ered snonard aql uI prmoJ Ilelap € sr (g) aotoq] 'aqlrrsap
ol pasn Suraq sr ,asaqo, pJon ar{l leqrvr 3w>1se s uoqsanb aql 'uo4sanb aql Ja,lrsue lou
saop ll 1nq'saleru roleJreu aql l€ql dSopue aql ol uoslr€druoc e palJar saop (3) aJIotIJ 'uoq
-sanb aq1 Jamsu€ lou saop qJIqM'Jaleal aql ol uonelal ul lalor{ aql Jo uolleJol aq} qllm sFap
(y) atroq3 .uaddeq lr{3T,o 1r q8norpp 'proivr e Jo uoqugap Tooqlxal E roJ pDlse aq nod p,rn
dlarey 'lxaluoJ uI prolv\ aql yo Swrearu aq+;o Surpuels.rapun Jal+aq e ia8 ol sreadde prorvr
aql JaUE aJualuas auo pue sreadde prom aq+ aloJaq aJualuas auo lseal lu pea5 'pasn sI pJorvr
aql ,.1 oq aas ol IJaqJ lsmu nod taqrunu auq e ualr8 are nod;r'p8ro; l,uocl 'azls e sa]eJlpul
ramsrre aq+ asneraq (g) asooqr dlsnoauorra lqgr-w nod uoqrugap aql uo dlar nod;1 'sreadde
,,asaqo,, proa aq+ q)Itl1v\ uI aql aq+ ol ratar ol atuq aql DIel nod leql sallnbar uoqsanb qq1 c'v
tious events that follow in this passage, as is evident by the narrator's understanding of the
misconception of the harmony he felt. If you chose (C) you may have recognized the names
from Greek mythology, but there is no mention of the history of the island by the narrator. If
you chose (D) you may have recognized the Greek names in relationship with a kingdom,
but the word "grandeur" is not the word that would be used to describe the island. IIt is its
natural beauty that captivates the narrator. You could eliminate choice (E) because nowhere
in the passage is there a reference to violence. Once again, you might have been able to
make the correct choice using the Process of Elimination.

8.D This question requires you to have an understanding of the vocabulary used in the choices.
Once again, you must remember that both words must accurately describe the speaker's
impression of education on the island. ln this case, choice (A) can be eliminated because the
education is a solid and practical British education, not superficial. You can also eliminate
choice (B) because it may be annoying to the narrator, but it is not archaic, merely adequate
and current. Choice (C) does not provide you with any choices that are accurate. Choice p;
may provide you with the idea that the education one receives there is inconsequential, but
it is not perplexing to the speaker. He understands that the education is adequaie, but he
is frustrated that it does not involve more of a romantic notion of education. Therefore, the
best answer is choice (D).

9.A The speaker finds the school constricting and the students exasperating. Th.y preferred to
talk about cars rather than poetry. He found this preference of theirs frustrating, so choice
(A) is correct. If you had chosen (E), you chose an answer that was close to coriect, but it
was more than disappointment that the narrator felf it was frustratiory as shown by the
way he developed his examples in a repetitive, almost sarcastic manner.
10. A This type of question posses the most difficulty for students. Once again,you need to look
carefully at the question and the choices you are given. Eliminate the choices that do not
supply the correct implications. It is hue that the narrator believes that there are shortcom-
ings in the British educational system. The education fostered an interest for science and
little for literature. If you decided that choice I was a correct answer, you are on the right
track. You can eliminate choice (C) and choice (D). if you look at choice II you will seJthat
the "myopic" (limited) vision of the students who only want to learn scientific information
and not that of [terafure, also makes selection II a valid statement. Because the choices that
are left all include If you have to look at III and IV to determine whether or not they also
apPly- Selection III may be true, but you need to refer to line 80 to make sure you under-
stand what the object of reference is to "mole-like blindness." You can easily determine that
this reference loosely applies to education. In that case, you can eliminate selection III be-
cause it refers to all inhabitants of the island. We do not know if this statement is true based
on this selection. Now you can also eliminate choices (E) and (B). Choice IV is not true, and
therefore you are left with choice (A).

11. C This question requires you to know some basic literary terminology. Parody is not evident
in this passage; you can eliminate choice (A). Aposhophe, which is often used in poetry, is
not used in this passage. You can eliminate choice (B). Repetition is often used by good writ-
ers, but choice (D) indicates that the attitude of the speaker is emphasized by theiiusage. So
choice (D) is not the best answer for this question. Take a look at (E). Hyperbole is not used.
Now, you need to decide between choice (C) and choice (D). This passage,like most literary
passages/ does depend on the author's use of imagery to make his point. So keep choice (C).
Thai leaves (C) as the correct answer.

I58 I CRACKIIIG THE AP ENGTISH TITERATURE EXAII


691 I SN0lrvNVtdXt 0NV SUt/r4sNV:l lsll l)tI)Vud

'lrarrof, (g) lnq ile /aseJ sl{} uI 'uoltJalas aql yo Surueau aq} a)ro;urar o1 dlaq sluaurala
9re
leqrt Suqse sr 1t-a8essud aql ur pasn alp sluauala dreralqleqrvr nod Surrse lo.r rr,rorl
-sanb aq1 'uo4sanb arfi ]e rool o1 s nod
Jo pa{se Brnaq sr 1eq^ Burpuelsrapun o1 ,{a>1 aq1
'suoqsanb asarD tn pasn uaag aleq
l€ql surral dreralTl arn IDIM JerTrueJ 1o,., .rn nod;r looq
aq] Jo )Jeq aql ol raJau dSolouru.ral
;o e3pa1mou4 rno,{ uo pazzmb are nod ,ure$e arug !I '9I
Ja SUe pAJJOJ
aql q (v) 'lQnop e lnorDIM 'alelrdordde '1r rage sauor +eqm pue auII aq+ arotaq
1 1sn[sauror
+eqM lP {ool lsnur nod 'uo4sanb aruara;a; e ur 'raqruaulay dlsnor,rard sr{es ra>1eads arp
}er{,1a
lou sI leqJ'sllH aq+ uI aruq srq,{o[ua ]ou IIIzvr raleads aql lErD salerrpur (g) acroq3 .parroln
q sIil 'sarnllnJ oml aql uaamlaq dltsotutue 8ur1aa; e saleJrpur (q) acroq3 .1.rJur"1e1,
;o
o1 azuodsar ruoDrJJns e iou pue
relap e q srr{J 'raarD rplridr.n "ql
;.1}eqr pale)ry,'
^.."4
dlsnor,rard rolerreu aq] pue a8en8uel >laarD aqr qlr^ sleap asneJoq (3) alernunla oqe
+r
ueJ no^ 'anrl lou sl leqJ 'uolsnlll ue se1\{ alerurlJ arp
leql sale)rpur aJror{J aqi asneraq (g)
alernrurTa uer no^ '(v) sl uo_4sanb ar{l roJ raMsue .pqur pue dpoq
tsasolJ ar{J uaa,ropq duour
-JPr{ Jo sSurtaal s,raleads aq} ol sJaJaJ
,,uorsnllr up se,vr }L, :auq rrunr"jn, .ql i.8roj 1,,.roq V ''I
ramsue lsaq aql s (6) arroq3'(g) aleuruqa u€t no1 dem
ieql IaaJ rulr{ a{eru lou
plp slFi aql'sauq1e luarledur i1a; arrer1.,{eru raleads aql q8noqqv'llaa se
13j a1lulu11a
ueJ no^ '11a1 ra>1eads aql leq,/vl1o alsoddo aw sl .}r .an4
'{ea^r
o) aJror{J a}€unurla uEr no^ lou
q sHI IaaJ turq aperu ,{aql salerpur (g) arroq3 'aJnleu rgr,r snoruorureq IaaJ ur1{
apeur daqg '1aa; raleads aql ap€tu sruq aqi noor{ ratur o} parse .rn no1 dlinlarec
aql pPaJ ol paau nod tanarroq 1uo4m1as slql uo paseq sJaqlo qlr,t,r alelnmruruoJ
l'**rl"l,
lou p1p
ra>1eads aqi leql ra;ur dlqrssod ppor no1'alrleJrununuo)un suearu
,,,Jrrro)eL,,(y) acroq3 (I'€I
'(g) sr JaMSue aq1 ,aro;a;aql .(d aJroqJ se asuodsa.r orues aql
saqdurr (g) arroq3'paprlord sralseu aqi ler{+ uoqerurolur Jur}uans due parno1ap s}uapn}s
aql leql salels rolerreu uaqm parldtul q (d a)roq) .luaualels anrl e $ arror{J srql .s3u,
9yl
-punoJrns rlaql alelf,ardde 1ou op sluapnls pue sralseru leql salerrpur (3) arroq3 .sub]lsanb
raqlo uI aleJn)JP se paztuSocar dpea.lp arreq nor{ leql stuarua}pls saleJrpur saJror{f, Jar{+o
aql
lE Tool V 'anJi aq iorruE) 'aJoJaJaql '1uaura1e1s srqJ 'uoqeJnpa qsrlrJg e Sulr,rarar pup
{aaJD
osle aJe IootlJs aq] uI sluapnls a{l 1nq's8utpunoJJns Jlaql ol uoquaile ,{ed
1ou deur s1aar3
aqJ '(g) Ioor{rs ur sluapnts aql Jo asoql ol }ser}uor uI lou $ puelsr aq} uo s{aarD aqt
Jo
apnlqle aql +eql 1vlou{ no^ 'pue18ug uI q ll se arues aq} sI soxenld uo
loor{)s aql lo araqds
-orule aql.an4 sr (v) aJrorlr rn luarualpls aql ]pql MouT op no^ .lrrral aql puelsraprm
l,uop
nod;r ua,ra 'anl1 are sluaruarers aw JI aas pue {ool p a{eJ 'uoqsanb
taaf,xg .rn r, ,rqi l"qi
Jaqruarual lng '(salrural Jo isau e) ,,friuywta|;o Surueau aql Mou)I .zr
l,uop 1,, des dew no1 g
QUEST|0NS t5_27
The passage is by Christina
Rossetti (78g0-g4),and was written. when she
was in her early thirties.
H,=:ry;|uTt#',ffi1,H:;ilf'*" is tvpicat or nossetu, who
was f.,.i*i,n iil-hearth
her
The Rossettis' christiria
and her brothers, william
of an influential mid-nin"ttt"it'-tt"iuly Michael,and Gabriel,
?*r.
urt, movement catteatrre pre-Raphaelite were at the center
Brotherhood. pre-
*?iJ;.','i:.:tTits.T.,Hq;lm,:.#f
Chrisrina's brother Dante (;g;"bry r***x'_1;T;,,fr ff::filce(o*en,^s.d
in. tu#"r;il;;; iiarhaerit
the truly cheeseball uttt movement) is gu'ry of one
1**q r1i* ;*uJ bante cruri.iii"rrJtti , *ir. died,
or ttur.ir"rirrl'"t of
the
FTi":L:i.T:'.',1J1'#ryU:"*mt!:ii1f""".il'Jthecasket;i,il;;Ah,,oves.;;
his poems back. The,rar, r*gh, n"ffi; up
is on Danre, whose r,rl*".tl"t^1111,d"q so he
:glriJ g;;
;,ff.,il:,i:ff fi :o1T".d;;s;"".r"g,",p".tio;"ilJ1[J#ffif:n j:ilu"r*HissiJter
any years spent
The poem on the test (like
almost ev.eryfhing Christina
Rossetti wrote) is a meditation
on the
li:T_r"ffi :lfrli:!:iF:T:?;tn*",:'m:*iti*iTi'"#:Effi ;fii,.,,...*.i.,
Although the bulk of the poem;t'-"ttg"s
the test lav several trqys i, u.."rribi" tl -ort readers, the questions asked
jor ti.;;;"ty. r.rdi"g;;; on
b.l,",ii.a.16
making assumptions tirat iit.rprgtir.,gpoety,ui o' guura
can't againsr
FF;;:#;;r';:".
on her a**'ti.J. ri"' shourd incoirect .riii.., ,r..r, sugges*he
;t;;;;;; that the p";,;;;;;i-,.
5iil:ff #[:H,T .o,,t._prutio,,
Ano,her**i#,i5,i?f :;f.T::*:ifr
tions ask about some or tnt ;ifffi **:*;[1{:xru;::,.,*,,f l;".,.
pou*aJ;;;; p"iitr. Thu.. ,* ,.**r questions, for exampre, about
the
;ti"'i::T:l]'tT;1,:::.1'!',T.1?$1:;:*:t*###;:poem,sri,,ui,tu-u co-pri.uuo.,,
This long-standing tradition
hopelessly old-fashioneddevice' "r
i""r[^irg with thi spidtuar forces of
the cosmos may seem a
important works using this
o*
** up to the p."ri^iJuy continue
tonutn,ioi."it e nioss"tti to create interesting and
logue with the metapliysitut
*o'td,l;;'k* 6.d;"ve1, not gnry has rhe speaker in
dia_
the soul speak with ihe matterJa degree further in
voice or tn"pu,i."ioilowingihe the iecond stanza byhaving
say:" the soul presents what raTil""rt.n what the past doth witness
trt. pJri n"rio ,uy ibout human and
that in this sranza thlnlst mortality. you needed to
is nor teing a1r..ttyp.u*"r.J'rr understand
not even being quoted;the
soul is inte?preiing'ti" purl;;il"
r ,p*5*. f";;il;r is probabry
r a tangled piece of rhetorical uil.n, of"rithe principar narrator. This
u.roi'rrid .u.rr", ,.,oriri.r"i.r.,ts
.ot',rt
overall, the passage, ,11.1
,og"rhJ*iin its queJons-" ,ii1. some problems.
work you wi' see on the Ap difficuliend or ir,u spectrum
Engiish Literature and Composition of
Exam.
16' D As noted in the
generalnotes to tn.
this, is ajgugh question.
answer choice (E)' five' But lu:11r.,- Most students choose
the past is notl. speaker. Thelast is being
principal narrator by the sour. interpreted for the
d"otrr*:ho,:^.. rt'rir"prriricated
is (A), one. The reasoning read-er, ,"fuiir"., pi.t
behind cnoosing
the soul, worrd, and^cojt"p*r""t ral ir rr,ririilt* o.nly the poet is speaking;
jrra
the poem is a kindof int
.r..r.,it,
ntir-o;;i"g*
il;'il;h," the poet. In this ieading,
thepoet sorts out her feelings about
death and the afterlife' ntit
i"i.rf*,r,ron Tis absoru*iyi,i*rrule
ryhi+
intend for you to think sh" (Rossetti certjdy did not
The problem is that it is
hua alturily r,"ra
an iitrrprttrtiir. rrr. "
.;;:J.t"" with the world o, coa;.
q".riilffi, ,,How
many speakers'"i*,does
the

I7O CRACKING THE AP ENGI.ISH


TITERATURE IXAM
'
lll I SN0llVNVldXl 0NV SUITYISNV:t lStI llll)VUd

'qluvr pauJa)uoJ sr'aloqn e se uaod aql Jou


,uoqsanb ul aql aql Jaqlrau
leqt sauaxue pnl1r1ds ro'luasard $ auou ararl,l- ssaq11 Suaas
raqlra's8urpeaJsnu snolJel raJJo saJroqJ paJJoJt4 aq1 'ssaro,rd 3ur3e prnleu arp to uoq
-dursap aroru auo 1ad sr a8eun aq1'dlucrroqdelaur Xlpr3p ol luearu sI,,derre au.RD 14 tl+orx
,TZ
e,,leqm pa{se alalr,r nod'dlprrseg'alqno4 qJnru ool nod uarrr8 aleq l,uppoqs uo4sanb aW iI
'(g)'ramsue paJJoJ aql qlyvt dluo ga1'a1qno4 tlJmu ool
lnoqluvt Jlasmod pulJ plnoqs nod dem srql8uT).Io lI ssor)'Suorrvr ra.&\$re aql sa{eu
sr-ql 'rurat e 3uo4s ool qJnru sI JeaJ ^'JJo
1nq'paleldrualuoc ,{ple1rar s derap prrsdq4 'parroJ
-ryq dpo s;.Zezrels ro; ,,derap pcrsdqd Io rea!,, 'larra,l-o11 'lJaJJoJ dlpuuelsqns sI € Pzu€ls
JoJ ,,aJIIJaUe aql Jo aJu€ldacoe aurocla./vr,, pu€ 'ldacle ol q8noua asolD sI ,,pqqaq Ual aq lsnru
ler{l plJo.rv\ dlqpea aql roJ e€1e1sou,, '1ezrte1s JoJ lel11 uosear plnotls nod'aldurexa ro;'(g) r4
'saJroqJ Jarrsue aql yo Sqprom aql ol uonuape asolc ded'suo4sanb Jo sprryl asaql q 'Suorrvr
U$s q parJo) sezupls aarql Jo lno oml lo Stmreaur aq+ Eag l€rll
raasue wq '&uotm r sa1€ru
leq/vr ro; dlryarer aJrorlJ ra/rslre qrea Surpear'gg4 asn ol se/vr da1 aql uo4sanb sgl uI iI
'02

Sqrt1 uo Jou suospas arp uo raqllau saerp 1eq1 ruaod eq1


's8tr.n1t

;o a8eur auo aql s ,,p1o8 dW.rl lsn5,, Jo aSerur aql'€I aql r{


'suos€as Sur8ueqr aql Jo pu€
,que1d dlprcadsa's8tqq1 Srql[ uo s/{eJp 1eq+,{.la8eurl
Jo asn a>leru ile sJalvrsu€ FarJoJlrI aqJ, J '6r
.uo4daouoc ruo{ sn qql,t- qleap Jo paas aql .,{rrer arvr toqdelatu 1ue1d raqloue asn o}
ro lqprq tuo4 luaurunm sr qleap teql asuas aql tn ,,uaTJI4s looJ,, a.re praua8 rn suerung pue
roqlne ar{I'a}H uEumq uI Uleap 1o aruasard FluaueptmJ aql q ol sraJar uolll4s-loor leq A
'p dgleap sI Jolprreu pdour.rd aqt luql 3q1ecpu1 se ,,ua{Jlrls looJ,, uo dn lcrd dpuanb
-ary orlm ,Euapn+s dueru ro1 auq la{Erualqnor} P $ I atnl 'aJII ueunq;o lftopafe4 reaupl
aw r{llm suoseas aql lo ssarSord prmdr aql lse4uoJ ,,'f,eJap dq1 plpqar lou lpt{s 'ua>Pr4s
loor 'noq; /:f,eyt1ul pnq pue Suudg u;lasdur aqtolr lleqs L, 'araq uoqsanb ur saull aql fl '8I
.lamel qlrm salenper8 SrmrmorJ ruo{
Jo uoqrpe4 dltsralnm Ielalparu aql
palrJap rural p 'aryatnalettuq.rnod pauJea aleq nod leq ueau pm aa#ap s,JolaqJeq e qllzvr
a8allor ruorJ uorlunper8 's.rouoq yo Stpleadg 'apatnal4aod sr al+1} 4aql laod puorluu aql se
parouorl are aldoad ueqrvr 'depo+ ua^g d4eod Jo poD uo4ed 'o11ody Jo ramolJ cqoqtuds aq+
servr (1arne1yo dlaueur e sr deq) pmq l€ql sr slaod suoquaru dlecurcads ramsue arp uoseal
aq1 .a1ed ppq s,Jesa€3 sngn{ 8uuer1 aas sdemp nod q}earrur leql q se 'qlearm Iarnel e sem
rila*Xp aql Jo ,,ppaur p1o3,, pqrSlro aql 'dAaod Io le^t'qrods u qdumr4 yo uoqruSorar
ur papJe/vre s€,rt salpal r{eq pue IaJnEI Jo puepe8 e '&anos ueruo1 pue {aarD }uallue uI
ra+PI aql yo aldurexa ue
s1 uoqsanb sr-q1 'aznrSoJar plnoqs Fnpl^pul ppar{lom e saJuaraJal luJuolsrq r{.rera11 asoql

+noqe {se r{lpuorsecro daqt pue'('ap'1a1dnol'lauuos'.roqdepru'apuns ''a'D urroJ


pu€ ruslJ
-qrro d.re;a1q 1o dSolouurJal Jrseq aql /vroul ol nod padxa daql'mool nod padxa uer daql
slaa1 SII r{)F{ speJ aruos ar€ araql lng 'arnprd ra8rel aql pup spelap aql qloq'pear nod
^
Ierralelu aql puelsrapm o1fiIIIqe mod lnoqe sr lsal aql;o luacrad &aqru o1fiq3rg'1,uop
nod ro lI /roDI raqtra no1'lsal aql uo suoqsanb a8palmoq arer dlalqqar aql Jo auo sr slr{l J 'tT

'sraleads rno; sluasa.rd 11


.quasard ruaod aql '1sa8
leq/vr JoJ d1du4s lnq uoqelardralur ue roJ 8uryse lou sI uoqsanb aq1
-8ns lq8nu ruaod aql leqm 1ou 's1uasa.rd uraod aq1 lpt{/vr uo s srsegdura aqJ ,,iluasard ruaod
22' c The line runs on from 7 to 8. This is another
terminology question. If it gave you any
you should refer to our section on literary trouble
terms for tdAF E"J;lil?"#;nd Composi-
to get rid oi tio* u"'*.rs
ff1ff:il*ii";Til:,Tff:,1:ffii3i vou are sure are wrons

23' B This is a question that many students get


wrong. Always retum to the passage. The
third
l h.,.ry*,
stanza presents a dramatic reversal
ery from the previous stanzas with an antithetical -*riirg and direction;t;ufig"ru.,g imag-
meanin!. In the first two stanzas,spring
and all the image-ry of spring are used to.tup..r".,iilrh;
energy, and life. you might easily
think then that-winteq, ui spring'r oppo.site, rwr.ru't,
haps (c) the coldness of the g.ui., that is, 1rj agi'g and ross of vigor, or per_
of winter nthe third stanza'in this stanzaGod
death itself. il tli. q"";;;k" i"i,n. meaning
says tharlow 'winter passeth after the long
delay'" \{hat follows are images of spring ttow
clJu.tf ,iJ to death anh ur spring
in the final stanza is a metaphor for the jJy of
,e.r.,ioi *irn coa. In the final "it*rir..
stanza, God
offers death as a joyous springlike o..urior,.
It is earthly life, separate from the Maker, which
is the long Winter.

24. C As with all questions with longer answers,


you must read carefully and eliminate when
an answer is partially correct. Partially correct
means all wrong. otherwise, the reasoning
behind this question is fully covered in the
expla;;;;;stion 23.
25. e fnferstanding the lines in question is not as much about the lines themselves as it is about
letting them make sense in the overall context
of the poem. If you understood the bulk
of the poem, then this question shouldn't have
been difficult. If the poem itself gave
trouble' this question might have as well. The you
incorrect .h;i;.r offei various m#readings
and over interpretations.

26' c one of the easiest questions on the test. This is


essentially a vocabulary question, but
chances are you were unfamiliar withthe
pusugJr ffi of the word ,,sptay.,,Figure out
the meaning from the context. None of the
incoiect ans'wers makes ,.rrru ir,'.or,text except
possibly (A), and we hope that befween (A)
and (C), you chose (C).
27 ' D You are certain to see a question (or two or three) like this one
on your test. If you got this
question wron8, brush up on your skills with
our section on gramrnar for the Ap English
Literature and Composition Exam (page 53).
As outlined in tiat section, th;;";;way to
figure out the construction of the kind-of sentence
ETS likes to ask about is to rewrite
sentence (in your mind-you shouldn't the
need to actually write it down) into a more
form' The sentences ETS ihooses are never nafural I

straightfor#ard "subject, u.ru, air*t-object, in-


direct object" sentences like 'Jack threw the
ballto me." The sentence that begins on line 24
"Arise, come away, night is past and lo
it is auy, rrrry rorre, My sister, My spouse, thou
hear me say," should 6. ,.*ritt rl shalt

"Thou shalt hear me say, 'Arise, come away,


night is past and lo it is day,
My love, My sisteq, My spouse.,,,
Notice we've put quotation marks urgTl-*hut God reports he will say. This is how the
sentence would normally be punctuated.
If you rewrite^it in this _u*6r, yo, ,f.,""fa be
to see that ',ThotJ,,is the subject. able

II2 } CRACKING THE AP ENGI.ISH TITERATURE


EXAM
tll I SN0llvNVtdXl 0NV SUl,!ISNV :t lSlI l)tDVUd

peal a^eq deu oqzvr luaprys aJeJ arD q4eJ


o+ pauSrsap auo ,sra.,u.sue de4
dlqeqord s1(q) a4oq3^.p*qr'q"r,dFyr^D .s{A[,uoreapqrre 1e Jo IIAa lsour aql
aq] +ou,arpnord .rCI ol sraJar
,,'palraduaq sl doqslq aqt ,1oe; q,,,tqde.{etnd"W or.r, .rJurr,rn4 ayJ .a3en3uel
al4darap
Jo asn raqloue s (g) arroq3 'o;ra,r s,uerud8rarr e se aro'nq bj o*grdl1pryr" u;iu^
Jaq Jo saSap,tud ,,,4tretr
II'J aql,,
spJolvr aql saas or{^, Japeal ssaraJeJ arr+
aleus o1 pauSrsap ,Ja^sue
de4 e s (y) arroq3 TEarJ uoquarur s,Joqlne aql sa{e,' .'JIAJ
,,'orpnoJ4 os ,qde.r8e.red
1op,,
luanbasqns aq+ olur uo4rsue4 aql pu' 'linrl.ro, uI pinf irn sanur^ s,rliue.rp .srl .ise4
+no
-uoc dq arpnord .srl{ aqrrJsa_p o+ pasn s1 ,a;yvr J
,,.ror""pqrre aq1 dpue.lD .s{Ai yo aldtuexa
aq+'rape.reqJ s,arpnord's.r1,1 Surqursap ol pa+olap
sr r{Jrq,r,r,r8nrrnd rqllo
ffi"r
aql uI J .20
.a8essed
arfi ul plalJe raq+rnJ uo.r; a8en8uel aarldarap
^
pue (v) saJlot{) 'a8er'rreru srl
lnoq' servr a8essed aql ieqi drouraru an8e,r aql ro
alel (f,)
saserqd aq1;o Surpear ssararpr uo paseq a.re ,,rr}sourop,,
(g) pue (g) arroq] .+r apar o1 rddeq se^'
'aJIM SIq o1 ra^od arp paraJJo ,s.ra11eu rnq
a^eq +ou pFolr aq J'sauop ur leqi salu1s aserqd aq1 .i€
c
Surql aq+ dq apeu asrou aq+ a{r' punos dleer
plmos saop ,,Jeln+I7,, pue'ptmos spro./v\
1,uec ll os,qra,r ro unou e rou s,lr,:iflT;5i;
qll,v' op o1 8r4q1ruos seq nrrodoln-ouo
s'/vroDl orl^ luapn+s e ol padde 3o{ +Eq}
lq8pu (r) a4oqr .spiorur,rirunl uala rouvr 1nq €urql aiues
aqr rpnru op (d pue (g) sarror{J 'uoqera88ex.
.tn lorr- sr (y) pue ,uor1era83exa ,' al0qrad
-dg'puels.raprm l,uom r*r"1r{rnr"i[ t{ilrn rer1*ueJ xpjnrrr.pr
r,uare o.{M s}uapn}s }pr{+
prozvr 8rq e ueql .raqlo 'lr
'ro; 3uro3 qJnru a^eq l,usaop (y)er10q3 .arueu uI q
airsoddo aql $ uoqenrs aqilpueqsnq rarl ra^o lr l€q^,r Jo
sprol oqrvr'd11en4ce ur ,arpno.rd .srl $
'qldap lea'r' ur sureldxa a'essed rq rn 1nq ,arpnor4 :rryrtl prol i ]r
;o aq1 ,aureu rn ,sr arpno;cl rcr .0g
fl
1ou'p;qrlezvr e o+ uolsnlt' pc€o1oq/ur e sosn (s) arroq3 'b)
se J1iJjt##ffi,1:rtll;
;o aldurexa ue palurod sE +ou sl +l lnq'o1 lnuqns o1 yooi ,rq rapun asoqr slradxa
'sr;41 Surqlaruos o+ sraJar arpnord
(q) arroq3:.rp'o4 .s;61 lou dp*r3 .rr4 ol sraJar
'doqqq p aJrM aq1;o a.raqds'ln*ron."w (g) arroq3
Jo ny"xri p*+*" i"qi^",r pue suoqrqwe s,arpnord
'srl J ol dlasop aroru qrnu $aJal €rrudtual
afqrw,lyy ,i1op .ruq pa^o) seq ror^eqaq
rrlodsap s,alpnord 'srl lvroq trl os saop lr 'arpno.r4 rc saqrrrsap q8noqlly
J Jo lxa+uoJ aq+ .67
11 3
aqi sl (v) os 'pa1e1s seq aq leq,,
U,'.,oI oqe'urser.res aruos
Jo alrsoddo aql uearu.+,usaop roqine aql lnq
pal'p
nJr"ri#rf::
aql yo alrsoddo aq1 iou sr dprelrar
1il acroq3 .buruparu papua+rn si Joqrne
+q8l.u auo lasop sr
rr +nq'nloqrndrq'il p*r1i.ro, aq lq8p
ro Jluorr sl sf{+ J} llguI ol raq ?o
+noqe pres q8noua.l,usr aJaql }nq ,aJIM ,,rirnrpqrrn"ri".D r*
'dpuerS 'srl{ Jo uorl,rsodelxnf n,{+ rl
'sqAJ
13iacroq]-."raq drroriou sr araqr ,aq o+ p*s sr arpnord "qq
leqll dlasDa.rd sr Buuaaunuop rr'nrrg:n;r^ g*r"n**op e Bulqpcsap a8en8uel
-eJn8rl luo{ pa^rrap ursrrcrnbollor n ,, (g) nrlo,{r '}eql arrrl
lsnt Bulop sqder3e;ed pra,ras spuads
uaql lnq ,,'arpnoJ4 srry lsure8e pJoM e aql'alq
o1 ,rorirrrlui dru jou sr ,sa1e1s
Jo+EJJeu aql .gZ
11,, V
'ppuasso st Burpear asol3 .uJaJsrp
ol dsea sde.r,rp
l,usr auoJ 'xeluds palsrlvrl Jo sruJoJ Jarfio pue ruotln8"., ard4rmu
qrru. ,po1n101uoJ +Err\,raruos aq ueJ
saJualuas aq1 tsal aqr uo aas o11de a.re nod asord
uerrolrl^rqi ,1rr ,.ra,r,aa,lo11:1sed luarar aql ueql dep
aq1;o scrdol aqi qrlvr paruaf,uof, arour su^ ,qazrou
ueuofrql duno, .1urrr, ,pue
'sla^ou arqs+asreg srq puo)as
lgglui ua$lr^ se^ lI
Jo at41,uamo1 ra$aqrnglalors ,ado110.r1duoq1"#;t;irs"rrna ,r,..
0t-82 sN0trstn0
this novel or itssequels, in particulTr,Fyml Parsonage,in which the rivalry of l\u[rs. Grantly
.w
and Mrs' Proudie is given substantial attention. It cert"ainly is not the authoi,s intention to
suggest a rivalry,.although he.may have intended to
foresiradow it. Choice (E) has some
merit' From the description of Mrs. Grantly, it certainly seems
as if the author favors women
who exert their powerdomestically and piivately. The
passage states, ,,before the world
she is a pattern of obedience; her voice is never ltud
... she know what should be the limits
of a woman's rule." Nevertheless, the ranguage in the answer
choice, ,,assert why women
should be seen, and not heard,'- suggests tlatine author
provides evidence for a position
stronger than the one he actually takes.

33' D Pity, answer choice (A), is best used to describe how ihe author feels toward Dr. proudie,
"her poor husband.'Although the narrator-may feign
an appearan ce of objectiaity, answer
choice (B), his opening comments make it clearihat"what
hl'presents is hiJ subjective opin-
ion' Given that, answer choice (C), emotional judgment,mightte
tempting, urrif,ir language
is shong enough to justify (D), sardonic condemnition
He ii certainly moJti"t nn proudie,
,
and his judgment of her does condemn her behavior. It
the Passage
-- pj
is choice --ae! is t|o extreme for
\-/ tnat

34' B We get no sense of Dr. Proudie's devotion to his wife


or of his moral compass, no matter
what we might want to in{er from knowing his profession,
so answer choice (A) is out. An-
swer choice (B) is supported by the text.of Ihe third
paragraph. Choice (C) and (D) suggest
a happy and loving mariage, not the picture painted
ly?rrir purugrupl,. (Ef -igt , describe
Mrs. Proudie's relationship to her husband, but not the
reverse.
35' C He is described as "aware that submission produces the
nearest approach to peace which
his own house can ever attain." Choices (A) refers most
nearly to a quality be'siattributed to
Mrs' Proudie' Choices (B) and (E) are.not
gupported by the text. Choice (D) is a trap answer
for those who read quickly and saw that the pasrug. liuu,
about the clergy and religious
matters.

36, A The maid in question has been un{aithful.to her duty. As is par for the course, on a single
phrase or word questiory primary dictionary d"fir,itiorr,'.hoice (D) is offered as an an-
Jhe
swer choice, as is a word it kind of sounds like, choice (B).
The other choices have no merit
whatsoever.

37. C The repetition of the phrase "woe betide" accenfuates


the seriousness of the servants, situ-
ation' It neither slows down the prose, as in (A), nor does
it satirize or mock the servants,
fate, as in (B)' The phrase is consistent with the narrator's
attitude throughout the rest of the
passage/ so (D) is incorrect. Choice (E) is too exheme.

38. B Even if you weren't familiar with the Victorian use of


"chatactet,, as shorthand for ,,charac-
i'
ter reference youcould derive the meaning from the
context of the passage-the maid has
been dismissed, and because of this "character," she
is unable to find dec.rit u-ptof-"r,t.
Choices (A), (C), and-(E) all prey on a reader's familiarity
with the dictionary dei#tio.,s of
the word, as opposed to theiontextual meaning. Choice (D)
is a trap for the careless reader
who sees "characte{' and "foot" near each.oihei in the passage
and overinterprets-perhaps
thinking that the footman is sent to escort the housemuia rroil
the premises.

174 J CRACKING THE AP ENGttSH UTERATURE rxAiJt


sll I sN0rIvNvtdxl 0NV suti4sNv :t Isll t)tI)vud

raq ol sarTdde rvtel aq+ slaal arls rvroq lnq'(1arvr se aldoad raqlo Jo 1o1e dlqeqord pue) lsquap
aql ol ropadns Jlasrar{ sraprsuoJ dprepar rarporu aql 'aleunuqa o1 q3no1 ool uaaq alerl
l,uppor{s (g) 'slsquap al}l[ ddaaro a)U ],usaop lsnf aqg 'u4q lsuleSu pasn)re se snlels s,ls4
-uap aql ploq ar{s saop }JeJ ur Jou 'deuvr sryl uoqenJls drnf drarra aJeJ plnom aqs ]eq] awns
-se ],ueJ a,r,r spr€pue1s p8al uErD raq+er umo Jar{ o1 Surprorre lsquap aq1 sa8pnf rar{lol l arp
asneJaq lsnf'(f,) ur'dpepturg '(g) aleuruqg 'aremeun ,,d1ap1duror,, raq a{eru l,usaop leql
inq'raq uo pasodrur saqnp leJrqla arp Jo atuos Jo aJEMErm aq deur aqs 'saop aqs asJnoJ JO
ilrrpra^ e ra^rlap o1 uodn pallP) aq ilyvr ar{s l€r{l ,/vtoDl lou oqs saoc 'dlera1q luarua}els aql
a{eJ ,,ealemetm dlapydruoJ,, ar{s sem'(g) uI d1cFl 1rq a{il{ e aJalvr sJaasup paJJoJw aaJq+
Jaqlo ar{J 's,Jaqloru aql Jo }eq} ol alrsoddo dlaialduror apnlq}€ ue saqlrrsap }I 'uoqernu4la
lsrry dsua up uaaq a^eq ppoqs (y) arroq3 'sarror{J aqi yo Surp,rom aql ol uoquage plrls
ded ol papaau notr dgryareJ saJror{J ra^rsue aql lardralul o1 nod roJ palleJ uo4sanb s1q1 A'W
'srualqold rnod yo (1p
ro) lsotu pa^los a^eq ppoqs,,{1ryarer [O.I8qsn 'pe1s poo8 e ol JJo aq plnom nod uraod aql uro{
q)nur s1t{l lo8 nod JI 'Jlasll lerJl aqi ol se Surpear s,aqs )ooq aql ol uoquaDe r{)mu sE sded os pue'urrq
uo sada sdel aqs ]uaruoru aql tuo4 dlpn8 tsquap aq+ spug dldurs aqs lsarlnp raq ol qreordde alqero
-uoq dpeprqred 1ou pue a8ue4s e sa{e} raq}oru aql 'lsquap e Jo IeI4 aql te drn[aqt uo salras or{lvr
(aldoad puoqJrJ pue lerrl leuorlJrl dlarqua ue alprreu lq8nu uraod aql'ralpour s,razr) lnoq€ peJ uI
sr uraod aqlleql arunsard l,upFor{s a,vr) .raqloyr[ palleJ JapereqJ E saqrrJsap uaod aql'dgeuuassg
'pqtu uI Eapr ureru s,ruaod aql lda{ pue uaod aq} Jo lualuor praua8 aq} Jo asuas
Sulrorvr e peq nod se 3uo1 se srualqord dueru oo1 nod pasner aleq +,uppoqs suoqsanb aq1 'gura,rg
razry udlore3 laod ueJrJaruv drerodrualuor dq ,,s,ropoq Jo aurT 3uo1y,, uraod aq] sr a8essed aq;
99-rr sNorlsrn0
'uoqeunulla JoJ
aleppupr dsea ue aq ppor{s (g) aoroqc 'q8noql 'surer8rda pu€ ssauasral q}oq 3u1te1 '8rn
-3ua1pqr aJoru apuru aq ueJ uorlsanb e sderu arll Jo auo ale sa)rorlJ JaMsue aso13 ';lasrnod
IrrT l,uop 'salroql ramsue aql qSnorql ssed 1srry rnod uo ll lda{ uala ro '(3) asoqr nod;1
'a{€ru a8essed prrroqdulau e +ou op sroqdepru rvray e 'se1y'arrqdursap dprepar sr a8essed
aq1 lra8uo4s lno suels (3) arroq3 '(g) aleunuqa uer nod 'azrrsnga Jo uoqrurJap ar{l lvrou{
nod se 3uo1se os'puoqorua dp.req sr lr'anqrafqns dpregar sr a8essed aql q8noqqy'cr1
-uepad Jou ssalJorunq raqlrau sr a8essed aql-slrmoJ qloq uo 3uo.r,u. sr (y) arroq3 'alaq dep
aql al€s IIIM god anrssarSSu pue'dro8alec Suorm-;pq'lq8p-geq aql olul IIeJ sraMsue raqlo
ar{l Jo }sontr dsrrcoddq raq Sursodxa dq uaql 'luaplrnba Ielros e ol lse4uoJ ul IsJIJ'smelJ Jaq
lno Suqurod dq dpa,l,ap raq Suqrour 'dervr Susnue ue ur alpnord 'sryr1 sazdleue roqlne aql i1 '0t
'qder8ered aql yo lurod ar{l aq }ou plnom
leqt lnq'qsa88ns (g) se 'Surrrq atul] preq E aleq lq31ur at{s qrn{M"'JJels ploqasnorl rarl ol
pueqsnq raq puodaq ll spualxa 11 'u8rar ulodsap Jaq lnoqe uoqeprads ralunoJ l,usaop ll
'a,roqe pauoquaiu se lnq'alrpe.41e aq lq8p (6)-uor8rtal Jo uoquaru aql-suoseal J€puls
ro4'(ado1g'rIN alrasqo ol papadxa $ auo'dre4uoo arp almb) alpnord 'sr14 Sutruasqo u4t{
'JIN aJaqa sr qder8ered snll se 'sluaul sll set{
Jo aperu sr uoquoru ou lnq'parnpo4ur sr adolg
oqe (g) aJror{)'arpnord'sryq 8u;aaururop ari} roJ uoqeruroJsu€rl e 1sa33ns qder8erud srql
saop areld ou rr lnq'padse snotSqar aqi Jo asneJaq Suudulal aq iq8ru (y) arroq3 'aurq aql
Jo lsal aql ,,sassarp rvrol pup uoqedrssr[p],, o1 uanr8 sr aqs q8noqlle 'Jallaq snot8qar;o
quod
a18urs slp ol sauoJ ll uar{r'r sraqlo o} salnr aql Surdldde q q aqs plJls moq Surivroqs dq os
saop qder8ered aq; dsr.rcoddq s,alpnord 'srl I ale4snlll ol q qder8ered aq1;o lurod aql J '6€
we don't know. Eliminate (E). This leaves only (D). Yes, it's fair to say that the mother takes
her responsibilities too lightly, and her certainty about the whole affair tells us that she has
no doubts about her fihress as a juror.

42. B This question should have been a piece of cake. You did not need to read too much into
the phrase "half-heard." Don't let the power of suggestion steer you down false paths. The
mother half-hears because she's reading. If the poet wanted to suggest age or poor hearing
she would have refurned to those ideas to make them clearer. Here, she wantJto reinforce
the impression that the mother has made up her mind so fully that she barely bothers with
the details of the trial.

43.D This is an exhemely tricky question. Many students pick answer choices (B) or (C). But the
dentist is not said to be a seminarian (a clergyperson). The dentist is uncomfortable, like a
priest without the white collar of that profession. The rest of the stanza relates the court-
room to a ship (e.g., the "plank," the "deck"). The dentist isn't compared to a condemned
sailor, but is described as though he is one when the mother pushes the fly-speck from the
page and says "she will push him off." This statement refers to the way in which she will
push him from the plank. It also suggests that she thinks of him as easily dismissed and as
insignificant as a fly, and perhaps as repulsive. But the poet does not describe the dentist as
a fly-speck. Choice (E) may or may not be true, but it is found neither in the stanza, nor in
the poem.0.ly (D)is corect.

M.C Here you needed to understand that the poem is about the mother and the dentist, not
about other people. That is, you needed to stay with the main idea. The phrase in question
refers only to the dentist; in fact, choice (C) summarizes it nicely. The dentist is theThackel,
wielder of pliers," etc. Yes, some of the items in the list are a bit confusing, but use your
imagination. How is the dentisl a'barbe{'? Well, think of the hydraulic chair you have to
sit in, or the bib the dentist pulls around your neck; aren't those things reminiscent of being
in a barbershop?
45. E Here you needed to stay with the main idea and not get drawn toward a silly answer.
Throughout the poem, the mother feels herself to be superior to dentists in general and
to this dentist in particular. The mother is a tremendous snob; she considers dentists to
be low lifes. Advertising is just one more thing that her-kind-of-people just don't do. You
might have had some difficulty if you didn't know the word "propiety." It refers to what is
ProPer or polite. POE should have led you to the right answer anryay,however, so long as
you saw that other answers all involved reading much too deeply into the passage.

46.D This is a super POE question and it should have been pretty easy. Choices I and II should
have been obvious. The mother finds the dentist guilty simply because he is a dentist, and
she then persuades the rest of the jury that he is guilty. Using POE you are then left with
just choices (D) and (E). So, does the poem imply that the dentist should have been found
innocent? Not at all. A11 it implies is that the dentist is an unattractive creep who drilled
a patient through the tongue. Does this action make him guilty? \{ho knows? Innocent?
Again: Who knows? We're never told with what exactly the dentist has been charged.

175 J CRACKING THT AP ENGI.ISH TITERATURT EXAM


lll * SN0llvNVtdXt 0NV SUtrr4SNV:t tStI il[]VUd

I '8uorm 1r 1aB {qeqold p,nod pue'aruq Buqse,,vr dn pua dlenlce p,nod Buorrvr
I uoqsanb
I aql pB l,uplp nod_g ualg 'aflras uoturuoJ pue drouraur,ro prrnq
Jaasrre oql lno arnSr; o1
dr1 pealsur pue a8essed aql ol oB 1ou o} spl^ a{e+snu sno4spsrp .a3essed
I {Jpq aq; aq1 o1
T)eq +ua,/v\ nod se 3uo1 os uoqsanb dsea dra,r e spm srql (.swra1,{rnlalrr
I
I paulJap a.re sldaruor wog 'ruaql uaa^qaq aruaraJJrp aql
io dressof rno ur
I ilal uer nod n*, n'1nr., pue ioqdupw
pue ara8 srural ar{+ augap uec nod arns a{EI :ure8e $ araq
[ lr 1nq dpearie salur] Jo aldnor
e 1r pauoDuaur aA,aM 'sI .roqdelaur e
Jo pqr
leq^.Jo eapr ue a^eq o1 papJao,,o,(,8uelods
sHl op o1 'aslnoJ;g) ,,'a1o4s1searq dsea u€,, q+rm speal Jaqloru arp aJaqm ,6I
aql Suriods
alqno4 qJnru oo+ pEq a^er{ },uppoqs nod 1nq ,alualalar auq ou sazrrS
uoqsanb aqj .3urpea.r
o1 roqdepru e saq)el+e uaod aq+ ararlm pu.rJ pue
o8-o1 sem uortsanb qp
lrnq o-1 da4 aqg A,.6i
'(oo1'araql nod sdlaq
q8noql) rDexa dV aql puodaq reJ ssaJrns pue arnseald sSurrq a8enSuel
11
-dop,raq'(pear nod aqa Btm4dra,ta pue) d4aod_aleprria" xpi ol rapro
ioy ,nn'.rr.r1 n s-
nod a8en8uel o1 dlr,rr1rzuas
*
do1..o.p ol lu'^,
Jo pupr aql +no lurod o1 pa1trn^ n^lng dlpu joplrrorrrrnb ,.rt
ra^rsue nod dlaq dlecr;nads srql Suaas saoq deld olrn aruoF-r"1n^
fuo40ura Stnsea,irul 'punos Stnseanu-sSrmrearu aar.ll
lprrro.{iar-lsqrF
il€ aJar{*r tpsrt,, to1asn e punoJ
serl Jazr) 'ruaod aql uI .asrJ ,seas
arfi uo ,nq *rolr'goilnoraa€ uB parJa
,seas aql ur se
aql o1-d1a1er.rdo'rdde aroru uala ro-ap4 Sursu e ur se '.ra1e.rnr Suruadaap
ot rcj4 o1
Jo qJaA arp osp sr 11 'uorssed ul puu pun:s ur aseaJJur ue sarldurr .1.r"*o_ "rroq,
,,sosN,, o1
palqs dpoa;rad 1nq'1xa1uor sFIl 14 qral Suqr4s/lensmm .rn r,11 "q1
,,lrrrriJ.ro*qsal aql"se,
aserqd atfi uI ,,saslJ. pJgM aql
Jo asn s,Jozry alou'1r pur; nod Jalalaqa. lno Bu4urod rprorvr
sr,{'rlsruB asneJaq lsnf pue 'sauqleprurs Buoly .qcadsord
luesealdun Jo aJeJ ,q1'.r, q1rro-
s/auo ur alsel ra+lrq e spus auo +eq+ :eapr aFls e;o Sulse.rqdar
qsar; e se srroM osle +r lnq
'sroqdelaru Jolres JarlJea aql ol {Jpq sJaJaJ ll,.auriq B,rr}se1, yo srw slas ,"rrjr-d1.rr,,
,\,roq lno Sqluiod qlrom s,ll ra+pmpas;o pr;q1rro*e qllr!\
a" p","8"*,
pue 1ue1d nq11in^ nr^
aq'uaql dgerr.roqdelary dlpn8 punoJ aq
III,' a_q lpqJ liarvr BuroS pL rrrr8irryl lnqir",ln-,
o1 tm8aq seq lsquap arlJ
'ralemerr r, ,,r*nq. :sdpq rtrelnqero,r ,ure8v .ezuels poor5,
roqdepru rye1d-aql-1le^-ol-paJroJ-rolres-pauoop aql samnluoJ "q1yo .g?
,,r.4rq aqg
Bo+rnl,, aserqd iI
'aturls
Jo sauoua^o 3uo'4s qlyvr'p'rynl suearu }I'dlprluassa ,llaM zuEaru snoJaqJal saop
ietllvl oS'(v) {ruo qlr^r Ual_ar,am,ilocl Bursn,snql .(g) aleurunrd';;h;,
ior,,.rorlnlh,;rl ro,
druearp Jo luaruoru e ut an8uo1s,1.tat1nd qq palllrp .1d
lsrluap aqJ s,araql lpulg'.(q) aleu
-lum'JEruuru € olrT aroru spunos oH rrpr{J aql plmorE }ue}srsse slti pasBqJ aH .oN
Zplu4i
lsquap ag1 s1 'aldnurrd peq-p = pPq-JIer{ aql 3u1snl1du1s dq a}ernurTa plnor no1 .af,ror{r
aql al€u.ru4la ol lrODI o+ paau l,uop nod lnq,lear8,mouT nod;1
isnonwa s,leq,M.qero^
aroru purJ nod (q) u1 .3ur1e1nr1er +ou sem lsquap aql,(f,) aleunurTa
uosear,*n,
roC '(g) alpunurTa os '8uorm s,leql .an8uol s,luaned qq g8norqi panlrp "ip-lprn*,
dlaler"qpp
o+ p'r{ plnoll }sl}uap aq} 'pailor aq ol (g) rog .ssan8 e rol ",rnq
(y) aireal p,nod +ueaur iror"qr"1
l€r{1w Mour },uplp nod;r'o5 'an8uo1 s,1ua4ed e qSno,rql lq31.r qIIJp aq }€r{l a^ol q}vvr
+r Jo
+no os Suroq srq +noqe oo+ duurry dlTrs Surylaruos s,araq+ pue ,sa,r.1aqs aql ruoq urr8 qpai
ralseld paplotu alqlvr rleqJ arp ptmore 1*lsirre stu Sursnqr dlerruor s€
uaas aq plnor +srl
-uap ayl leql_des o+ (V)ul luearu snuatpal lerlm MornI o+ a^eq
no1.peqJp
l,upp = peq_JIEr{
;o a1d'tmd aq+ pue irocl o+ JIaEr spuar 1nq1,rorlr"nb n yo a1d-e"" pooh i orln ,,1i .aldrue
-xa poo8 e sr uoqsanb snD pue lsar aql olur drepqero^ aruos Buoleaus yo
xl^ n'inq sJg v .r7
50' D This is primarily a t-erm question, but you
could have arrived at a correct guess without
knowing that "poetic jushce" refers topunishment
that reflects the crime.lFor example, a
counterfeiter,buying an expensive old painting
with t;g"r money only to discover later
the painting is a forgery') In the seventir that
stanzZ,tnu po"? a"rcribes the dentist now
position of a patient, gnpPing the arms of in the
his chair and being most uncomfortably
This sort of reversal atso rils under.the category drilled.
p""ti. justice. If you were unfamiliar
with the term poetic justice you could haue i-rtiu"J "r iiu p.rr*uy
good guess by reading
the question carefully. It refers to "Mother'r
tr.ut*L"i;if the dentist. In which stanza is ttie
mother most directly involved with the dTlif,
himseu (and not simply the legal process)?
In the seventh stanza, where she "strapped him
*r, to arru him awuli;'rrri,
should have made (D), the most attractiuu g.r.rr. "ria.rrtanding
Guess.By the way, there,s another more
technical definition of poetic justice, which"grs
witt prorauty not use. we cover that defini-
tion in our glossary.
51' c There's our old friend irony again. In the
phrase "Mce Mrs. Nemesis,, the irony is
delicate; in fact, it has almost iecome irony's not very
nasty little brother, sarcasm. To answer
question, it helped a great deal to know that ,,nimesis,, this
a is an archenemy. 0n the poem,
Kizer actually refers to the Greek goddess,
N.-erlr, *no represented righteous anger.)
If you knew that a nemesis is an a"rchen.*y, o,
negative (which you
could have figured out from context), you could"uu.,lusisomething
rtuuJ..uro"ed tha-t,,NIce Mrs. ,something
nasty"' contains the kind of contradiction that
makes for irony. Barring that understand-
ing' you should have worked with the terms
yot, k r"w and,rr.d pOE. All the terms in
answer choices are covered in our glossary the
of t.r_r.
52' D These I,II, u questions are made for PoE.
A{ter reading through the items, you should
have gone back to the final stanza.and reread
it. Then to'ot ut the items again. \{hich choice
is easiest to decide upon? Choice II should
look weird-eir-inut it. The stanzadiscusses
the mother's idea of God; voltaire is an afterthought,
anJ ail that,s said is that she finds him
"indispensable'" I{hat voltaire's views.le,
the pJem doesn't say (and ETS does not
you to know voltaire s philosophy).wilh expect
item Ii go.,.,.noi..s @)'and
what about item III? social decorum refers to pori"t. G; u.e go.,e as well.
uurt"vror. In the last stanza, the mother
mentions that God instructs in 'hygi9n3 and
deportm.r,ii' thut is, in necessary social graces.
Item III is a keeper. Even if you did"n't know
what decoru- meant, which makes more
sense: the mother believes God agrees
o. disugre"s ? rf yo:,ug"t A+;;rar drift of
the poem you should know that ihe Mother lith h91thinks God"shu.es her views. Keeping item
means you can eliminate (A). All that,s left III
are choices (C) and (D). okay, teys.toot
item I' Here you needed to read closely. she at
says coJir- i"aispensablei; r6uit gooa
enough to justify the.'deeply held" part of
item I. \{hat about .,unsophisticated.,, Is it
sophisticated conception or fut. oi"i"e to think a
that God cares about hygiene? Not really.
Furthermore, Kizer's comparison of the mother
to ,,true iJoh,urr,, reinforces the
unsophisticated idea. Does the mother think
she's ,trrroprur,i.uted? Not at all! She thinks
she's hot stuff reading voltaire and all! But
the q""riio.iao"sn't ask what the mother
of herself' It asks about what-the poem says thinks
about her. In the final stanzaKizer has some
fun at the mother's expense. Iternl is a keepel,
which makes the right answer (D).

I78 I CRACKING THT AP ENG[ISH TITERATURE


EXAM
6ll a SN0tMVtdXt 0ilV SUi/rtSt{V :l lStI t)tIlVUd

'no[' palzznd (g) arroqr ur


,,a)uare^rqlup,, ruJal aql Jr ua^a Ja^flrp +q31, atR
nod uapo8 a^Eq plnorls gOcI os ,a1equ41a ol dsea dlpld uaaq
a^eq p1not{s pue ,paqBsnf aq
ol arua4xa oo+ are lEq+ salqs puoqorua dn
1gr IIp saJrorp +JarroJlr aqJ .lla,ra ,rr11"r*nq1'3*
-pnpuoJ dlpnpe aJe uaql ,*unlrrnsrrT_qrs
Jo Jarprau 1nq1d"oa rq1 pr'rn aJrI ,y-ol ,,1rp.rrp
aql 'pnpuoc radord Jo asuas dloous s,Jarnoru aql
ol srayard razry lnq'8w>lroqs uaaq a^€{lqep
Jo qselJ r.11 *
,o*n"q Bull.raprm'rq1 ,",
pasnrue,(f,) ot
dn sppe Srmrosear srll Jo 11y'saruinbazuor rr8erl "bunn+t.r ".11'arualelrqrue
nrtnq 1,.r#op apnrp+e JarrEAe) s,Jaq+olu
aql os 's1a8 aq ler{^' sa^rasap oq^ daa.rr e a>lrT prmos saop
lsFuap aql lpr{l1rny ,q1 dq paua
-uos $ Srmn aloqrvr arll 'l€JrlrJJ dnq8l aJe sluaruarers asaql .r+lap
aqr Jo uorsr^ s,Jaq]oru
aql salelal aqs uarl^ rou pue
,,'srsaruaS _sJlry a)IN,, se Jaqlou aql ol sJaJaJ aqs uaq^ lou
'os dp.r4ua lou lnq'raqloru aqr ol Jpaqi"a-rr r,'/;y
,.rrf.rp"p*turre; [s,rarporu aq1] 1o
{JaJr\,, aln an)sar or a4e+ro^ uodn qlec aqs se uala (arueS0rre pue) ql*rar r,rrq1o*
qlrlvr passardurr s,aq5 'uoqentls aql qlllv' unJ serl .razp1 "qt
'dpso14i 'i""*si"lsassed ,Jnr.,+r,q
lr s^oDI razry dlqe*uoqe pa^eqaq seq Jaqpur aql spJ'pue+s
IeJrqla Fuorlua^uor dg .an4
s,ll puv 'lsnuap e 3u1aq roJ ueru e SrmnuapuoJ rr rreItm pnn qrr".1d11ard
Suraq r, ,;qro*
aW Sqlqql gas.rnod punot dlqeqord nol ,*aoj nqipnrr
noi ,y .uo4sanb auot p q sHI iI '99
'pauoquaw Jalau aJe slarJaq snor8rar s,lsuuap
aql iramsue paJJoJ
aql $ qlrM uar are nod 1u'pua aql
t'(iuopsanb raarxg rrn rr rryl raqtuaura.r) alernrurTa
ol sar4l asoql Sursn saJroqJ aq1 q3no.rq1
{JoM ol sE,/vr op ol peq nod 11y derue uorlsanb
srql a^e8,,iJaJlreJ qlra paqlnour-ua11o^l
luaued srq lvrou p,ry 7...rrrrol,qlqeq puos.rad
'arue.readde ,aJrlJeJcI /,uorssayord ,,n4q dq pauoop ,paruoop ,paruoop
,;^;il;r:"fi aql V 'v9
'alnJ leL{l ol suoBdarxa
arB letD sarualuas lnoqe suoqsanb ol sDIII srfl 1nq,an4 s,leql
1se
arp ol lxau pareld aq plnor{s srarJrporu l'rn
4lpo* daql (s;p.ro,u
lq8nel.tnaq oqn xlqnfl-d ,",r,o1 .1,.rfp xrgl
uaq/r lr sa{ll sJg .raqloue auo o} asolJ aruoJ o} a^eq
lou op qJa^ puE pafqn5 .nod ,vrorql
-"-lT
*r^T-1] 1dans,, pus,raqlol/{,, uaom+aq aualralrn sprol,4,leralas terll q uoqsalb
,q*o,,,ain1'drp;i;;;'{;#d;6iffiffi;;?#,":Jffi:i,il'#;,:#
1lllg l'+"o
aql uaqa 'auo sFIl a)rl suo4sanb reuruerS0pnasd ro ,reunuerg
{se ol sqr srg d11rya.rer
aJualuas aql ppar pue ruaod aw ol
{Juq auoS aleq ppoqs no^ raq}ohtr sr ra^sue aqJ e,,Jno
1darvrs,,^oqar $lse lI ,flJerrseg .uoqlanb pir*Srrp n ln .frnr*l
,ffn
lrrf
reunuer8 e l,usl dgear 1r'IJEJ uI 'uousanb"op""qrra*-
.reuure.rS dsua dpa e uaaq a^eq pForls srqr g .gg
HOW TO SCORE PRACTICE TEST I
Section l: Multiple Choice

-(1/+x-) =-
number correct number wrong Multiple-Choice Score

Section ll: Free Response


(See if you can find a teacher or classmate to score your essays using the guidelines in Chapter 7.)

essay 1 essay 2 essay 3 Free-Response Score


(out of 9) (out of 9) (out of 9)

Composile Score
x 1.23 =
Multiple-Choice Score Weighted Section I Score

x 3.06 =
Free-Response Score Weighted Section II Score

Weighted Section I Score Weighted Section I Score Composite Score

AP Grode Conversion

Ai cTd" ,,

107-150 5
t
93-106 +
a
73-92 J

43-72 2

042 1
U tsol olllroId
tSlm z rstr illl)vud

snorqn^ aq' ua^earr


"d:Xt^'fi"if;:n " 'aprm
Jllv+waad','-d;;'ffi:l:]'.T'jfi:':i'4ru";
+noqe punor rvrou sJa^rraasorl'
uopuor q araq l! q"gr*;
"ririr,.rnqrnq
lros re8jn,r aq,"in+.r'oy "q1
uro.r; a8en8uel rno pasueap "pn*-
aaeq.daql.,fro,i"roy 1gp;
prn
"'Y"i:ff[,1":H ;::r: "q, o+ x'",,J"* iJ1n,*"q,
aqseN seurordl_ .rno;o slaod ;i; il;;;tJ#^T,H;'ffi ,,?#:;";
*'
-::!i"1,""'.i,"J:;il::Hffi;$TuJj:#:"iii , , '#ifftH'ij#:ix#:f#,:.,",r:y#"J:,#f:if
Iailaq sr rpnui +r ;1n.ii;;;il*rffffi"i? oD s -ry^11+.r, r" ,",r'u .i"r araq+ toalaq^ se ue Lrlaod
.
ar'l srq .' ?:-.T:f
ueql €snec s'auo
nasurrq qro,oi
o+
,"q,,ufr:;il;*";il;
,"d^n1t*3+ ft".il.rrqr;;"rr',J;,di:i$I;J:fr,1,
ra-{n lsn.' o^ ,,^J.r'au.rr"raq Jir'"# ,r.,
lletd uj 9^eq o+ raded
qr'^ ^aN,, atrsea,r
r:JifrT f3,11$l:,15 "q, ,.,",, "o
=r"s"J vJ "1"!a x'"^
'p"o* "
j:J: j#+",Hr,Ttr#Jf#:,:::',{"fl#,,,,"T,lrffi :*il:ii*#::{*}:;'"'i#
,,q

"*;;il:H"1
rllr*fi'd;r:rrffi"#:;fy' .' ;#;:ll:if[#3j+ii?:"#*g#'''
:;l{3#*;i1ffi'"'"'*rh,l'lffi '""ff y,sn'*;i;**fi '":,,il3
.".**lll,ti#i:"ft
;t.x1;';"fl#:'"i'I,o.,,,,i',*il;;:Jffir."$ #S:i'.{""THi3,"'ilf:i3i""Ii'i:l#ffi
'Tot t rno arou sril arots t.?"ro.,
o,,
,rnu,t,lJ,"fit ;:|fi:i:;Hl,",yll:it4 "q ;;i;;s#t"o
_ -ro_parrer
Lpns e "o au
sdaax ul
xru"a,j,"q,,,"-r"d$###:,T.f1ij#t#jjf '"' -',1h5tr;#,'{;;F;#::,i:t:ifrffi ;:
j:-t1" sreJlrd tarur aqr ro3 'a;o;41 ;;*;;;
o',:::0""; oJJ"y:::q.::ro.{*N'il#::j*:i
ipasno.lretql"i,t_::3.r:""*r".1"s"#"rsq,
,

";:j':;fj:"'x5'i*{1*}il::+';'x'ffi':H;'u
,.;;,1l" *n1n3'n nJilj:iT":',fffi1'":ilffX
'"
wlil) os)
plro^'{"'nd;#:.ti-i".r
"',",T*'Jfi,1:g::"t?:i.{:"0"ot*r;,rilll.**r
ol q,eor;[il;;;Y
idifiH:il:ill,'
,,*f;lli:::ff#:df"f,#3"|.iit*?Y"u,r,
,
'a^p ur ,r,oo]'in,u.nru'r
,#;1f ;Iffitffi'rf,?1:_ _
'll,!ot'n'u'nlu'r t.q l', \p au!., sd;liil:F:#Xiifr"#llXT
'ou o:?jo,'";il"::"1ffi:,'l1fjjil,f.:"*,:f
nn su!pnn"*"Lp,,o^ q1"pn i*i:t
0"" ;,,,;::1:
aloq^A e g+asrdsap,suad_4aw;q;il;;:;paep
, t"o;:;rfrITJ::"Ti"lITj"*"'n
rvt
o,a.,,
"'aqr
e,',-;;,. ]3T-n_?

"'JT"p;i',oq
l NlaodJo sa*uaua aqr qll
"^""qi"

4
srq a^eq o1 adoq :qiJ"l, q3ry .__^ - -^ saluraug
dr1ao4;o ^^*yu
.{ureyur
",t^:,-P.f"
J.ia ,"tte lsure8y a.ruJa^'J uv
"H:'i":'#,l'J:"T:jiIl'lH,",T:*r*r[r*:"::,:
,,,, n,_,".T,".T:j"xTI,,,:H,"##"r^:r.:&T{ffi
.prro
Burpuods;
rauv ard+so*L"'''"'llll;J1fi1"Jil:fiil'Jl::l"H"r::jj#'JH:ff#":,il:i;fff*Ti,,'"""Xx;1ffi
rnoq l_arurJ

I NONf,iIS
NOf,tISOdhIOf, CINV
AUNJWSII'I HSI'IDNII
1. In the fust paragraph, preachers are accused of all In lines 34-39 London is described as
the following EXCEPT (A) flooded
(A) plagiarism (B) a damp, rainy city
(B) stupidity (C) the main influence on the English language
(C) dullness (D) a cultural garden
(D) eloquence (E) an important port city
(E) laziness
The main idea of lines 39-48 is which of the
"Satumist" (line 9) means following?
(A) astrologer (A) People are motivated by concern for their
(B) nymphomaniac reputations.
(C) depressed and depressing Person (B) Poetry is fair to the virtuous and the evil alike.
(D) pagan (c) Poetry is inspirational.
(E) foolishly optimistic person (D) Poetry is most atlractive to atheists.
(E) Poets are very judgmental.

Lines20-27 argue that


7. Who is Salustius (line 49)?
(A) poets must take second jobs to make a living
(B) most people don't respect poets (A) A French poet
(C) there are too many poets (B) Sidney's nom de plume
(D) poets have to work hard to present consistently (C) The Roman god of poetry
fresh material (D) The King of England
(E) poetry books are never bestsellers (E) The Wife of Bath

"New herrings, new!" (line 24) 8. What is Bath?


(A) refers to an implied comparison between the (A) A state of sin
writers of new poems and the sellers of fresh (B) Acharacter in Chaucer
fish (C) Amarriedman
(B) suggests that poetry is slippery and hard to (D) Apoet
catch the meaning of, like fish (E) Atown and spa in England
(C) implies that poetry is just another commodity
(D) implies that poetry grows stale rapidly, like fish 9. In the last paragraph, poets are said to be like
(E) compares poetry to rotten fish
(A) lawyers
(B) mayors
(C) chronographers
(D) townsmen
(E) angels

I84 : (RACKING THE AP E}'IGTISH I.IITRATURE EXAftI


98tl z lsil I)tllvud

uo+IUt (tr)
o+eLI (C)
snqsnlps ())
dauprg (g)
aqse11 (y) slsturyes (S)
: slooC (C)
o1$gssarau,slaod yo anssr srallrm pea6l (J)
aql o"rr"r r."1fTff .n,
sralrralearD (g)
sraqceer4 (y)
r__suoruJas
azr.re€e1d o1 (g) aqD ,,sauI Ip,, arp
ltFneaq pue a+e4s.nlll oJ (a) Z(9
lpr{r\A .ZI
snornellrl nrfl ,rnrr"q*"
o, /r\
.-^:?:d*r aln &ina
snory4l aq1 a8ernorua r; iri
oJ (V)
salLuaua s,,,(4aoa (A)
stueo4 (q)
spEIIPg (f,)
'uraao11ol "* r" u"*ft?x1#r"j'fiT3;fpa; r, rorpne aql (A)
qao4 (y)
i,esofi,,
suoulras (S) Jo +uaJaJar aL{+ sr +eqr ,Z
aull q .II
acuerr,alar
1qj
aqdo.rlsode (g)
gi uoperalrye (q)
"*"J;X (v)
uoquadar
^dpolnq aruarpn€ rr€ . xopered (3)
eradaoleruouo (g)
*-io,a !.o ;;il i;Jn"r,ff ;HiJff:.##""jff roqdepur (y)
,,
;o aldruexa ue q 0I aulf .0I
Ouestions 16-28. Choose answers to questions 16-28 16. The situation described in this poem is
based on a careful reading of the following poem by
john Donne. (A) the end of a romantic relationship
(B) death
Let me pour forth (C) the separation of lovers
My tears before thy face whilst I stay here, (D) the end of the world
For thy face coins them, and thy stamp they bear, (E) a pleasure cruise
Line And by this mintage they are something worth,
(5) For thus they be
17. Lines 10-16 are an example of
Pregnant of thee;
Fruits of much grief they are, emblems of more- (A) paradox
\Atrhen a tear falls, that Thou falls which it bore, (B) dramatic irony
So thou and I are nothing therL when on a diverse shore. (C) metaphor
(D) metaphysicalconceit
(rc) On a roundball (E) dramatic monologue
Aworkman that hath copies by can lay
An Europe, Africa, and an Asia, 1g. Line L9 is an address to the
And quickly make that which was nothing, all;
So doth each tear :
(A) moon
( rs) Which thee doth wear, (B) world
A globe, yea world, by that impression grow, (C) poet's soul
Till thy tears mixed withmine do overflow Thisworld; (D) workmen
by waters sent from thee, rny heaven disolv'd so. (E) beloved

O more than moory 19. To what do lines 14 and 15 refer?


(20) Draw not up seas to drown me in thy sphere;
Weep me not dead in thine arms, but forbear
I. The speaker's tears which reflect the beloved
To teach the sea what it may do too soon.
II. The beloved's tears
Let not the wind m. The beloved's clothing, which has been torn as
a symbol of her grief
Example find
(25) To do me more harm than it purposeth; rV. The ocean, which is salty like tears
Since thou and I sigh one another's breath, V. The rain on their faces
Whoe'er sighs most is cruelest, and hastes the othels death.
(A) I
(B) I and II
(C) I, II, and III
(D) I and IV
(E) All of the above

20. Which of the stanzas do NOT include images of


roundness?
(A) Stanza 1
(B) Stanza2
(C) Stanza 3
(D) Stanzas 1 and 3
(E) None: All of the stanzas contain images of
roundness.

I86 T (RAC(ING THE AP ENGTISH TITERATURE EXAM


rtll u lsil l)[]vud

prmor8 aql (s)


areld luaragrp y (a)
adorng ())
IIAH (s)
uaA€aH (v)
dSololeqrsa (S)
eu€alu (0 aqD ,,aroqs asralrp,, saop lBqM '72
uqlallered (q)
.(uor1 (3)
lus€Jres (g) ,,uo41sodsrp drrtms,, ;o alrsoddo aq1
aloqraddq (y) 's8ur1aa1 ddder4m yo arrqsaSSns sp uooru ar{J G)
uooru aq+ ur u€ur aql Jo alelTlot aqJ (c)
Jo asn aql q8norqt pa^olaq aql sral+Bg uooru ar{l;o adeqs prmor aql b)
ra.4eads aq1 'rueod sr-rp Jo srogdepru papualxa ar{l uI '82 saprl s/u€aJo
aql pu€ uooru ar{l uaauurpq dlqsuouelar aqa (s)
sJaAoI Jarpo >Iaas ol l.{8F aqJ G) ssappoS e;o ramod aq; (v)
lau8 .raq ure.4sa.r o1 razvrod aq1 (c)
aql (r) 20z-6l sarnl roJ uollurJoss€
s+ualuala lern+€u aql aJuanlJur o1 rartod
aler.rdordde ue ION sr 3uuvro11o; ar{l }o qrlqM 'EZ
urr,q ilH o1 razvrod aql (s)
+Jeaq srq >leaJq ol razr,r.od aq1 (v)
quau4uoJ at{I G)
z9z-02 saul ueuDlJo/r at{J (c)
tn pa^olaq srq ol aqrJrs€ raleads arp saop $qM 'lZ plro,|vl aql ())
il€qPrmoraw (a)
8q1aay alq€nle^ e sr;arr8 dw (A) sardoS (v)
a>1es rnod ro;
eJeJaJ //qJr-qA// prorvl aql saop l€q1vl ol'tlaLqIwI 'ZZ
are daql asnecaq Srr.nparuos rl+Jorvr are sreal dru (c)
Srmlsa.r;ar dleuopoura are- s.rea1 dtu ())
aJ€J Jno^ lf,alJar sJaAoI
daq+ asneraq Stln4auros Vlro,r,r are s.real dru (a) arn uaart+aq puoq 1enxas aql 8uusa88ns Ja+Br\ (a)
.sJa^ol ar{l
srual dru Jo lles arn rllrom 1ou a.re nod (v)
Jo ssaussalq+py Sqg1qs aq1 3u4sa33ns ralerr (c)
se paserqdered aq lsaq ueJ t a11 'gz 'pua ou aleq salJJrJ asnpJaq'sralol ar{l
Jo uonm luaueurrad pu€ f,nusoJ arll aroJarar{+
uearo (g) pue'a1c4e pa;red e 8upsa83ns ssarrprmor (r)
uooyq (q) 'uorle,redas ,sJaaol
sulof, (f,) aql Jo saru€lslp ls€^ aq1 8unsa88ns aqo13 aq+ (s)
saqolD (g) 'uoE€nlls /sraaol aql
srca1 (y) ;o ssaussaladoq aq+ 8ucsa88ns ssaussapuom (v)

2uraod aql lnoq8norql paq€r ;o sa8eun parn€lsns s€ paqrJJsap


-sns $ d.raSeun yo saddl SurmolJo; aq+ to qcTrl\A 'gZ aq dlale.rncre lsour ueJ ruaod snn u1 draSeun aq1 'rz
Ouestions 29-39. Choose answers to questions 29-39 (50) Among her forebodings of disturbance, not the
based on a careful reading of the passage below. The least was with regard to Caesar. Caesar was a veritable
passage, an excerpt from a short story by Mary E. Wilkins hermit of a dog. For the greater part of his life he had
Freeman, describes a woman about to be married after a dwelt in his secluded hut, shut out from the society of
long engagement. his kind and all innocent canine joys. Never had Caesar
(55) since his early youth watched at a woodchuck's hole;
Every morning, rising and going about among her never had he known the delights of a stray bone at a
neat maidenly possessions, she felt as one looking neighbor's kitchen door. And it was all on account of
her last upon the faces of dear friends. It was true a sin committed when hardly out of his puppyhood.
Line that in a measure she could take them with her, but, No one knew the possible depth of remorse of which
(5) robbed of their old environments, they would aPpear (60 ) tkrs mild-visaged, altogether innocent-looking old
in such new guises that they would almost cease to be dog might be capable; but whether or not he had
themselves. Then there were some peculiar features encountered remorse, he had encountered a full
of her happy solitary life which she would probably measure of righteous retribution. Old Caesar seldom
be obliged to relinquish altogether. Sterner tasks than lifted up his voice in a growl or a bark; he was fat
(10) these graceful but half-needless ones would probably (65) and sleepy; there were yellow rings which looked
devolve upon her. There would be a large house to care like spectacles around his dim old eyes; but there
for; there would be company to entertain; there would was a neighbor who bore on his hand the imprint of
be Joe's rigours and feeble old mother to wait upon; several of Caesar's sharp white youthful teeth, and
and it would be contrary to all thrifty village traditions for that he had lived at the end of a chain, all alone in
(15) for her to keep more than one servant. Louisa had a (70) alittle hut, for fourteen years. The neighbor, who was
little still, and she used to occupy herself pleasantly in choleric and smarting with the pain of his wound, had
summer weather with distilling the sweet and aromatic demanded either Caesar's death or complete ostracism.
essences from roses and peppermint and spearmint. So Louisa's brother, to whom the dog had belonged,
By-and-by her still must be laid away. Her store of had built him his little kennel and tied him up. It was
(20) essences was already considerable, and there would (75) now fourteen years since, in a flood of youthful spirits,
be no time for her to distil for the mere pleasure of it. he had inflicted that memorable bite and with the
Then Joe's mo*rer would think it foolishness; she had exception of short excursions, always at the end of the
already hinted her opinion in the matter. Louisa dearly chain, under the strict guardianship of his master or
loved to sew a linen seam, not always for use, but for Louisa, the old dog had remained a close prisoner. It
(25) the simple, mild pleasure which she took in it. She (80) is doubtful if, with his limited ambition, he took much
would have been loath to con{ess how more than once pride in the fact, but it is certain that he was possessed
she had ripped a seam for the mere delight of sewing of considerable cheap fame. He was regarded by all the
it together again. Sitting at her wihdow during long children in the village and by many adults as a very
sweet afternoons, drawing her needle gently through monster of ferocity. Mothers charged their chjldren
60) the dainty fabric, she was peace itself. But there was (85) with solemn emphasis not to go too near him, and
small chance of such foolish comfort in the future. joe's the children listened and believed greedily, with a
mother, domineering, shrewd old matron that she was fascinated appetite for terror, and ran by Louisa's house
even in her old age, and very likely even ]oe himself, stealthily, with many sidelong and backward glances at
with his honest masculine rudeness, would laugh and the terrible dog. If perchance he sounded a hoarse bark,
(35) frown down all these pretty but senseless old maiden (90) therc was a panic. Wayfarers chancing into Louisa's
ways. yard eyed him with respect, and inquired if the chain
Louisa had almost the enthusiasm of an artist over were stout. Caesar at large might have seemed a very
the mere order and cleanliness of her solitary home. ordinary dog, and excited no comment whatever;
She had throbs of genuine triumph at the sight of the
(40) window-panes which she had polished until they
shone like jewels. She gloated gently over her orderly
bureau-drawers, with their exquisitely folded contents
redolent with lavender and sweet clover and purity.
Could she be sure of the endurance of even this? She
@5) had visions, so startling that she half repudiated them
as indelicate, of course masculine belongings strewn
about in endless litter; of dust and disorder arising
necessarily from a coarse masculine presence in the
midst of all this delicate harmony.

I88 ff CRACKING THE AP TNGLISH LITTRATURE TXAM


68tl Z lslr llll)vud

'u1t4'ol s,eslno'I
Jo sanlel I€uoqrp€r+ lsaqSrq
aql eqr.rcsap daql G)
Sutreuaur (A) 'snrua8
StnPoqa'ro1 (6) Jrlsrup passardar s,esrno'I aleJlsuouap dn 11 (O)
arusuad (3) 'ssaullr l€lualu s,€smol ale4suoluap daqr (c)
lueu8rPur (S) 'araqds l€rn1lnJ
I€crrlt€s dPua8 (Y) aurunual dlpuoPlPe4 e ur uo4drosqe
sI (7 puP €
palualuoJ s,€smo.1 ale4suoruap daql (A)
'saulap +sa^req I€Jnr
sqder8ered) resaeJ;o uondrnsap ar{r Jo auol aql '7€
luarrue Io slen+rr ssalalurl aql azrloqurds daql (y)

uorsnTle (g) Z(Z Pue 1 sqder8e'red) sarrrlrne


uo.rorudxo (6) plor{asnoq s,e$no-l saqlJrsap lsaqluarua}e}s qcTt{M '0s
aPuns (3)
roqdelaru (g) u€urom 1arur'qsulas e sY (a)
z(uorr (Y) u€uroM luaJouul'Parallaqs e sV (C)
ueurol!^' qsqooJ'paualq8rr; e sV ())
yo aldurexa uP sI €7 auII ul,f,71md,, prom aqI '€€
ueurom qsIPIIqJ'arueu e sY (g)
ueurom SurraatmuoP'ragrq e sV (V)
saluor{ drquac-qluaalauru uI pooJ lad 1eorci,fi sI (a)
uI '62
slruraq ur dreqrlac a8ernocua zpazlralJer€qJ esrno'I sI .{!(or{'srural IIEJa^o
ol luearu aarleurd snor8rla'r e;o ped q (C)
uoqrsodsrP
a8ueqa u€J pu€ la1p dq pa+JaJJe ar€ sJounq pue aae*reur erm'eorddB 'raq yo lq8noqll"#X; "t"
,{ppoqleqr Lroaql,(m4uar-qluaapqu e spagar (3) aldurrs qq Su.nlcrmru 8op p1o aqt le palool es1no'I
luaunlsnrnd su;o Ped sr (A) 'sauoq puE qsau Jo lalp ,{reurn3ues pue Surleaq qlrzvr
dlra,rod s,eslnoT sPagar (Y) radural snora8uep srq parrl Jelau pu€'sa>1ec pue qsntu
-uJoJ Jo aJPI Jparse uo rur-q pal aqs 'runl reau oo4 oB ( g 1 1 )
(7 qder8e.red) lalp,,cpacse,, s JesaeJ'zE o1 lou aldoad paur€M sdervrp aqg dlporal s1t{ uI qlp}
leer8 peq aqs IIFs lraq qilrvr a11ua3 rfiarr sdezvrle servr
a^oq€ aql Jo auoN G) aq pus 'raqloJq peap Jaq o1 pa8uolaq per{ eq asn€Jaq
a^oq€ ar0Jo IIV (o) €bp p1o atlr to puoJ d.raa;lasraq s€ra aqs'qled qq ut
AIuo A b) Surpaalq uarpITtIJ lueDouuT lvres ar{S 'a8elpn papren8un (p11)
dluo 111 PUP'II'I (a) pue ppb aq1 q8norql a8eduBr aql uo resa€) Jlasraq ol
dpo 11pue 1 (v) parnprd ar{S 'auo uI pasry dlalaldruor arotu aq pFoqs
suorssassod pue slsaralur Jrar{l uaqrvr's^€p asaql Jo
$lJrrl lrlau 8op p1o u€ qJ€al 1,uer nod n auo'lou plnom aq leql adoq a1++11 d'rarr peq €srno-I
sEapl parnurcxaun ,,'lno trrrt{ a>lel ol Buro3w; $91)
pue Sur8ueqrtm dq padde4 aq ryD aldoad 'AI dep aruog'araql dn pap urr-q daal o1 1am lqBrruzvrop
roradura Superrul s,lr pue,, 'des pporvr ar{ ,,'uMol uI 3op PaJn+eu-Ja$aq
ue dq paranbuoc Suraq a1q sr Srndrretu 'm € l,r4B alat{J,, 'sl€^ra}rrl 1e r{1qmo1 almb ralleru aql
AIaIDOS r4 uorwdo srq Surttmouue 1da1 1nq'palsrsap aq leql
Iarur € Jo Jol€Jrpur ue fl
sleurrue o1 fi1anrc '11
paruPF os zvrarS €srno'I'asool uml ps o1 paldruaBe (001)
'
a8ueqc auroJlam o+ aJrl paqlJJsurnf,4J uaAa pu€ €tmrrer"r;o rolu€IJ Uos s,€smol;o alrds
raq 01 pawolsncJ€ oo1 trrvr.o.rS s€r{ €slnoT 'I uI'peaq aql uo ury1 pa11ed pue urlq o1 dn dl1ue1p'r
apo4s aH'sP,lvt aq se uln{ /v\es'ssaupMalqs pue asuas
+eql €epl aq+ acrot
aq1 paJorunq-poo8 qq qlrrvr'.larrarvroq'ao-['snoturoua pue
-urar o+ a8essed sgl l4 pasn sI r€sae) 1o d.ro1s
dppep pDIooI pue saqllno radord umo sr-q $ol (s6)
.I€ "tt8nrt
aq lBql os 'umr pamop€qsJa,ro uorlelnda'r sru'paurcqc
Z:I1UJ ar€ sluarua+e1s Surrvrolol aql Jo
qrlt{11
35. The contextual meaning of "sanguinary" fline 117) is 38. Which of the following are accomplished by the
(A) expensive Caesar vignette?
(B) feminine (A) It shows us joe's down-to-earth, kindhearted
(C) masculine character.
(D) vegetarian (B) It syrnbolically shows us Louisa's fears of the
(E) bloody future.
(C) It serves as a slrmbol of what happens to those
36. Judging from this passage/ which of the following who refuse change.
best describes Louisa's beliefs about gender rela- (D) It provides a humorous satire of small-town
tions? concerTrs.
(E) All of the above
(A) Men and women naturally belong together.
(B) Men and women should remain separate.
39. In line 46, how is the word "indelicate" used?
(C) Men bring chaos and possibly danger to
women's lives. (A) To indicate the differences between Louisa
(D) Women help to civilize men's natural wildness. and Joe
(E) Men are more intelligent than women. (B) To indicate that Louisa considered her thoughts
inappropriately sexual
cl. The contextual meaning of "mild-visaged" (line 60) is
(C) To indicate the coarseness of foe's personality
(D) To indicate the inferior quality of Joe's
(A) having a calm temper belongings
(B) having a gentle face (E) To foreshadow the vision of Caesar's rampage
(C) having an old face
(D) being confused
(E) having a kind mask

I9(l : CRACKING THE AP ENGTISH TITTRATURE TXAM


--

l5l: u tsll t)tl)vud

u8rsap adecspuel G)
ru€arp Y (cr)
suer.rela8a,r yo sualqord ag1 (f,)
qsad uapre8 Jo uoqermurla aqJ (a)
II reM plroM (v)
.0?
Zaqrnsap dleralq ruaod sr-ql saop teqM

deru rze1rtr ppb aq1 prmo.rSlaprm passeS (pg)


uaesun arp ol paluasuof, ile p,{aql {I*o Jt
'daa1s dru ur Iarreq aql Suoplq8gs 1
ruearp I'urroJ dn-padumq slq ltmr{ I lq8Tu IIV
dep rage,,(ep.ra1;e dep dpear pue pe>IJor aru
sdaal aq'zrto11a; dll^ ptO 'Ual >IJnq) auo s,araq; (gg)

'qlrmquoJ a8els uo aru€f, raIID{ adalm.eq aq1


.roqdepru (s) 'p,req dn asoJ aur aprsrn Jarapmru aql
ursrrul (c) aaJql-oml-auo O '+xau dqeq raqlouy
uorourdxo (J) 'pr€qJ ssu s Jo J€aI e w pa{oor{ ilns
uo4rsodelxnI (s) qlaa+ alpaaulaq'1a; ^Ir€a
pue 4E aql ur peddogdW @Z)
xope.red (v) aqs larpour eql paddorp I ralel se+mmu uaJ
Jo aldur€xa poo8 e sr (9I aql) ,,saqard treln rvl^le1,,, 'EV
'saso.r SurreaqJa^a aql rrr umop palp aH
'aJeJ s,TJnqDpoon +sal${ aq+ uo peaq € MaJp 1vtou
'aIqeIJI+sn!
'8unlI>l lo; sarlard uer-ulmJecl qlmt pagnd
q 8qilIT auos ler{l s1sa33ns ruaod aq1 (S) acer8 ruol; ualp; lsg1ced pasdel e 1 (SI)
'JelJllrts All€puassa 'sasou leau /Eallnq aw'zz' at{l Jo IaaJ a{l ol
sr ef,ualorl IIe leql qsaSSns ruaod aql (q) Surmrw lpnoarySu'pres 1'sqlnoru rno ruoq pooJ aql
'sJaIIaq snolSqar se
IeuoBBrJr pue 1euo4orua
se are sJarlaq pc4qod 1ertr1 s1sa33ns ruaod aq; ()) 'slorreJ aql Surpeaqaq's+ooqs rtoJroJq aqt Surddru
'rusnn^^J€c
qoled alqep8aa aq+ ra^o {ool uaq+ pu€
lercos lrerodrualuoJ pue dqdosol.nld asrnoJ Jo Jailsur e se splo8uetu agg uzvrop lqSnoqt{e\a @1)
rzeN uaam+aq qalpred smerp uraod aqg (g) 'q4erJs o1 dn sn;o il€ 'r{clors aro+s-a}els pue
'uoll€unuralxa
salp.re8n rno JoJ a,vr' ueql apruedc at1+ roJ
unural Jo Jrrolaqr rzeN arp qroldxa ruaod aql (y) asrom ou'ure8e dn paurn+ daql Stmrrour 1xa51
aIISTVC sr Surznrollo; aql Jo r{rlq A '27
'a8uer;o +no luaruaspq-qns-qns e peq ddqr rnq
'auols8urppnd qlpn lnqs paruoqaoqs slrxa qloq (g)
'lq8rpre sezvr ruaql lsure8e peq ar\ aseJ aql pve aurl
u.nprr\ qe4ualod ar€ uoqn)asr"o oXXt"fii* G) auoq aql 1e prnb'IryrJJaur se parnleal s€.&,!.
lualor^ llprluassa sr plrom aql (c) a8uer;rxg ure.r3 pue p€C aql ruog quoq lno)JorDl aql
aruapuadapur '1q8rr 1no urnl +,uprp $lJnqJpoo^A. aql SursseS
o+ Ip4uassa q pooJ uzrto rnor{ Surqe.r (f,)
aJrI
^ep^Ja^a (s)
Jo sal$€q aq1 lsure8e luep8rrr-.rorra aq lsnru azvr MolIoJ 1eq1 suonsanb aql
plJorvr arp JaAo 3r+{n+ are sl€rurue aril (v) ol sra.^A.sue asoor{J uaqt /u.nxn) aurx€trAl f,qs4cru4cpoo74
se paqrrJsap aq lsaq ppor,r, ruaod alp Jo auaql aql 'I7 pallFua uraod aql peau 'tg-0-T'-ffipsa-fr-o
"vr,o1aq
44. All of the following statements accurately describe
47. Whichof the following statements describing
line 4 EXCEpT the
setting of the poem are true?
(A) The legal rhetoric of this line reminds us of the
I. The stereotypically peaceful garden is
perversions of the legal system.
.. an ironic ru-ttirrg for'the violeice
(B) " Airtrght"
^listoJicglpuns
on its legal mea_iing and its described in the poem.
literal meaning in the context of glssing.
(C) The pronouns in this line establishL,,.r, II. The garden syn6ohzes nature anc
thereby enriches the speaker,s allusion
against them,, mindset.
to Darwin.
(D) The line proves that the speaker,s attitudes are
correct.
m. The garden makes this poem a pastoral
(E) The aural closure provided by the end rhyrne Poem.
echoes the sealing up of the woodchucks,
rV. The specific references to the garden
provide a realisfic setting in which to
den and the closed mind portrayed in the
consider the serious issues raised bv
Poem. the poem.
V. The garden is a sy.rnbol of threatened
45. \Atrhich of the following best describes the tone of civilization that must be protected from
the
poem? encroaching predators.
(A) Righteous outrage
(B) Helpless sorrow (A) I, II, and III
(C) Ironic satue (B) I,II, and IV
(D) lndignant protest (C) Ut and V
(D) II and III
(Sl Quiet triumph (E) All of the above
46. What is the most important thematic point made in
48. The word "Nazi,, in the final line of the poem
the final two lines of the poem? is
(A) If only (A) a metaphor
the woodchucks had all been killed, the
garden would be safe.
(B) an allusion
(B) Even garden-variety violence is similar to the (C) a simile
(D) a paradox
atrocities of the Nazis.
(C) If only the woodchucks were all dead, the (E) a metonlrm
speaker could sleep better at night.
(D) If only the gassing had killed the iaroodchucks. 49. The phrase "beheading the carrots,, (line 12)
is an
the speaker would never have had to example of
con_front the violence in his nafure.
(E) If only the gassing had killed the woodchucks, (A) ametonym
(B) metaphor
the speaker would not have had to see their (C) personification
disgusting deaths. (D) anthropomorphism
(E) s)rmbolism

50. "Y'in this poem refers to


(A) the poet
(B) the father woodchuck
(C) the narrator
(D) a Nazi
(E) Darwin

N CRACKING THE AP ENGTISH TITERATURE EXAM


t6lI z lsil r)rr)vud

CI]OI lnIV NOA -IIINN II NOIJf,IIS


.OS
NO OD ION OO
.NOIIf,ES
OCI OJ
,CIg'I]Vf,OI
SIHI NO XTIOM UNOA )JIIHJ AVI^I NOA SI AruI lIUOdlIg HSINIJ NOA dI
I NOIDIIS CO CINtr
dOIS

,ITI,II,I
A PlrE,AI G)
AI pue'il'II'I (c)
IIPIrPI ())
AI PU€ III (s)
ApueII (v)
'uraod aql sazrJ€ruuns lI A
'r.uaod
aql Jo uolsnlJuoJ aql sMopPqsaJoJ
dlprruo.rr luarualelsJaprm EI 'AI
,,'letQttrr,,
pue,,alrlJalla,, riloq u€aur o7,,1t131t,,
pJoM aql;o dlrn8rqure aql uo suJnl lI 'm
'3uo,rzvr
'uolsr^ urealp I€aJJns e JoJ pue lq8rr;o uoqsanb aql sasreJ lI '11

waod aql to apn+{Frlslral arp suopu€qe lI G) 'al4erJ€u Sumsua aql seruerJ lI 'I
'ursqeluauruoJr^ua
euorprmJ uraod stql Jo au1l lsrg aql saop rvroH 'Zg
lnoqe 1u1od leroru 3uo.4s p sa)eur lI (a)
'ruaod arp Jo lsar aql slJlperluoJ lI (J) 'uorsntuoc s,raleads
'Strgaay sr raleads aql aJu€I€qun
. Ieuo4oura aql aleJlpu saru^ril lupls aql (s) aql sal€4suouap au{ ar{l Jo ssaussalasuas ar{I G)
. 'rusrJseJ sasJopua JolEJreu ar{J (v) 'craodoleruouo sr ar4l aql Jo uoperallle aq1 (q)
'suorlJe s,ra4eads
;uraod arp Jo ezuels aql to rorroq aql sazrseqdrua ,,JalIDI,, unou aq1 (3)
Ieug aql saqrrrsap lsaq SurzvrolloI arn Io qrr-qM 't9 'su€runq Jo alor drolepard p.rnleu
aql sazrseqdura ,,ada4aneq,, arulcafPe aq1 (g)
'Suqerado
loqurds (g)
uorsnge (q) sr raleads arn qJrqm raprrn sapnlple
tmd (r) IErn1Inr aq1 yo dlqerclJlue arp sazrseqdura
uoiourdxo (g) ,,rDIMquoJ a8els uo atueJ,, ase.rqd ar6 (V)
roqdepur (y)
zluarual€ls
e s€ suorprmJ (f atlD ,,lq8p4e,, prom aql 'gg qrlq^a dq paqrrasap lsaq sr VZaqlJo uolptmJ ar{I 'Ig
SECTION II
Total Time-2 hours

Question 1

(Suggested time--4O minutes. This question counts as one-third of the total


essay score.)

The passage that follows is from The Secret History by Donna Tartt (1993). This introd.uction
sets the tone for the rest
of the novel. Read the qas.sagg carefully. Then write a well-organized essay in which you
characterize the narrator,s
attitude tow-a1d Bunny's death. In_ your essay analyze the liteiary techniques that the'author has
used to portray the
to the passage. '
narrator and his attitude toward the events that followed. Be suie to include specific references

The snow in the mountains was melting and Bunny years I might have imagined myself to be somewhere
had been dead for several weeks before we came to else, in reality I have been there all the time; up at the
understand the gravity of our situation. He,d been t9R bf the muddy wheel-ruts in the new grass, where
Line dead for ten days before they found him, you know. It the sky is dark over the shivering apple biossoms and
(5) was one of the biggest manhunts in Vermont history- (45) the first chill of the snow that will fall that night is
state troopers, the FBI, even an army helicopter; the already in the air.
college closed, the dye factory in Hampden shut down, Wlu, are you doing up here? Said Bunny, surprised,
people coming from New Hampshire, upstate New -
when he found the four of us waiting for him.
York, as far away as Boston. \A/hy, looking for new ferns, said Henry.
(10) It is difficult to believe that Henry,s modest plan (50) And after we stood whispering in the underbrush-
could have worked so well despite these unforeseen one last look at the body and a last look round,
events. We hadn't intended to hide the body where it no dropped keys, lost glasses, everybody got
couldn't be found. In fact, we hadn,t hidden it at all everything?-and then started single file through the
but had simply left it where it fell in hopes that some woods, I took one glance back through the saplings that
(15l luckless passer-by would stumble over it before anyone (55) leapt to close the path behind me. Though I remeirber
even noticed he was missing. This was a tale that told the walk back and first lonely flakes of snow that came
itself simply and well: the loose rocks, the body at the drifting through the pines, remember piling gratefully
bottom of the ravine with a break in the neck, ind the into the car and starting down the road like a family -
mucidy skidmarks of dug-in heels pointing the way on vacation, with Henry driving clench-jawed through
(20) down; a hiking accident, no more, no less, and it might (60) the potholes and the rest of us leaning over the seats
have been left at that, at quiet tears and a small funeial, and talking like children, though I remember only too
had it not been for the snow that fell that night; it well the long terrible night that lay ahead and the long
covered him without a trace, and ten days later, when terrible days and nights that followed, I have only to
the thaw finally came, the state troopers and the FCI glance over my shoulder for all those years to drop
(25) and the searchers from town all saw that they had been (65) away and I see it behind me agairy the ravine, rising
walking back and forth over his body until the snow all green and black tfuough the saplings, a picture tl-rat
about it was packed down like ice. will never leave me.
It is difficult to believe that such an uproar took I suppose at one time in my life I might have had
place over an act for which I was partially responsible, any number of stories, but now there is no other. This is
(30) even more difficult to believe I could have walked
Q0) the only story I will ever be able to tell.
through it-the cameras, the universe, the black crowds
sprinkled over Mount Cataract like ants in a sugar
bowl-without incurring a blink of suspicion. But
walking through it all was one thing; walking away,
(35) unfortunately, has proved to be quite another, and
though once I thought I had left that ravine forever
on an April afternoon long ago, now I am not so sure.
Now the searchers have departed, and life has grown
(40) quiet around me, I have come to rcalize that while for

I94 II CRA(KING THE AP ENGLISH LITERATURT EXAM


96ll Z rslr l)tDvud

s8ururumt .a .e -
.ralem Jo sse13 e dlprder Tuerp puv .o1ods all
,,Zalnru aq &raq[ Jo aJro^ erTl lleqs u€q+
pealsul parp daql TqnD ol dols lou p1p daqf
.ralq8nels Surreor aq+ o+ srrorl aTrI par{snr oryr
puap dddeq rlorar{ asaqt ueql lW OI)
-ln€aq aroru aq plnor +eqrr,r' flneaq;o 4e1 dqrt
urn8 dq qso8 dq aa8 dq o8urf dq
drro8 dq aureu snorrol8 rnod urrelJce suos rtql

"",Tff,i:1"TJifr;_1::ff Lf,""T,H,,,
oB pue aurof, salJn+ual Jo sr+, [,4rmot autT
dur d1.rea s,u/vr€p aq1 dq aes nod uBJ des
qo rluoJ os pu€ ,sur.r8pd;o puel nod a,ro1
I E)rJaur€ po8 asrnoc yo ol lxau,,

'rusrlor4ed;o lrlall s,tuaod aql seqrurexa 1eq1 dessa paznre8ro


-IIa$ € alurvr'uoBcrp pue'auol'a1d1s se s+ueuala dreralq qrns Suuaprsuo3 dgryarec ruaod 3qmo11oy pLnU
"W
('arocs dessa p1o1 aql Jo pJ.n{+-auo sB slunof, uopsanb sryr 'salnurur O7_aury palsa88ng)

7 uo4san[
Question 3

(Suggested time--4O minutes. This question counts as


one-third. of the total essay score.)
Lr some works of literature, mothers or the concept of motherhoodplay
central roles. Choose a novel or play of
literary merit and write a well-organized essay in wirich you discuss
tire matemal interaction between two characters
and how that relationship relates to a larger theme represented
by the work.
You may select a work from the list below, or you may choose
to write about another work of comparable literary
merit.

ADoII's House The loy Luck Club


The Awakening
Medea
As I Lay Dying
Mr s. W arr en's P rofes sion
Beloaed
ARoomatithaView
Black Rain
Pedro Paramo
Bleak House
Pride and Preiudice
The Color Purple
The Scarlet Letter
Daniel Deronda
The Seagull
Dombey and Son
Sons and Looers
FifthBusiness
Sophie's Choice
The Glass Menagerie
The Sound and the Furu
Hamlet
The Stranger
To the Lighthouse

STOP
ENDOFSECTIONTI
IF YOU FINISH BEFORE TTME IS CALLED, YOU MAY CHECK YOUR
WORK ON THIS SECTION.

195 I CRACKING THE AP EllGttSH |_TTERATURE tXAil


QUEST|0NS l-15
"An Invective Against Enemies of Poetry"
is excerpted from pierce penniless, His
supptication to the
Deuit, bv Engtishiatirist
lhgrnas
Penniless in about 1592' Although
Nu'r" irsoz-roor j. a lou;";t**
much oi his work *lr"rJ
ilffi;ffiil
uy,lorrrnuo,,rr*
be considered reactionar y bigatry,
Nashe is admired for his enerletic u"a ""*
,utuiirr.t;.m'ol;ilnding prose style. In the passage
t;::il?,fff
tr::TJ'"ffi yJlii:ff ffi ,:Xtl"'i"-r"*ii";rcon-tr'ibutio"io,oJ",v,especiary
1' D "Eloquence" is the obvious answer
here, since it runs contrary to the meaning
sage' (And you remembered you were of the pas-
looking ror ** .;.,i rary, right?This
"except" questions.) All the oth", is one of those
u.r*"rs straightforward acc-usations in the text,
for (E)' which is a little obscure. But when _are except
Nashe
tuirc uuo.rt the preachers, ,,quarter
mon"'he means a sermon given once a quartel, ser-
only four trmes a year. The implication
really do-n't work very huta,'"nrii. po.is, is
as the r,.*t pur'gr'uph goes on
lll-fr:fl..achers
2. . be tle pranet of depression and groom,
ffiXT:j,:*:1",e;1s,,3i:l y:,":T:shj l" and
i;#ll*:::f ffifi?:#:ffi :ii""ff ;:,
;!*;i*H*'r'?ffffi
"o-?:0"':-T:",gll?lyr.."*,J.iffi
liilL'n1::"#:.::1lyt"::':i:gltri:Ff Irrrrc ulan lt ls
that saturn is;;i;;;,; jump to ,,astr 'fffr'#ix)
:::;Tl"t: iX f:3*1!"-:;o1tlu
::::13'.'^",,i;91:":',nrlllly;i;:$d"1il*iJT?lt?.#'-*;:T:#,:';lJ
jg;;a;ffi
l#*,yfi:':i"y::::):"*g::iri:it"i ffiff ;:il:i#:J'";
up,"u.hu..(E),,opt?mi,,t.;;i,#;i#ii::i#,:
ffItJrff
ff the required definition.
of
":?lf#::'*tiJnof
3' D unlike thelazy,nlisr.alizing
preachers, poets must have new material
public won't buy their books. (A) is wrong all the time or the
because the various professions
these lines are only implied comparison, presented in
io, u poet's .u*., not a_list of moonlighting
(B) is wrong because Nashe's auiience poets.
who present stale, recycled material.,(C)
do"1t:jd;;;; poers; they only reject bad poets
and @f are
but might tempt those who think in terms ffiru,ury irr.i"uu'itoih" purrug",
of ,r,urt competition.
+. A Nashe is comparing poets to fishmongers, who must "tpiu.l
constantly keep their product fresh
the marketplace' He is continuing his"thought for
about poets being superior to plagiarizing
preachers. You courd make argurients
in gupport of (B), the idea.that poetry
meanings' but Nashe hasslippery
thaq he;s del."d#;;;;ry (C) and (D)
esting ideas' but they :-lt-lr*l.t
are not in thii passage, and are in"fact
are also inter-
contradicted in other portions
of the reading, which praise the special
citural
just ridiculous and completely "r.uni"g;;iil;;;J*"oiilr.
contradicts the meaninj;f ,n. el *
passage.
5. C Nashe is asserting the p,lrity and beauty.
of the English language, and argues that
as the seat of literary c,rltute, influences London,
h"*.Effiil ;i"A
i; ail over the country, like the
streams of a fountain spreading outbeyond
the iountainit*r. ri..ur*., p""o
prove English in London havei national
influence. Answetrlor, (B), and (E)
*i"r*-
rect associations with.the watery imagery are just incor-
of fountains follr lempting because it includes
is the souice of luliure, but does not'include
the-importar,t conc"pt or
ffi:XT llffLt:}]*

I98 N (RACKING THE AP ENGTISH


I.ITERATURE IXAM
661 I SN0llVNVldXl 0NV SUI/trtsNV :Z IStI t)tI)VUd

'luexa aql aroJaq


Tooq sry]
u dressolS aq+ ralo pear ol arns aq ppoqs nod'surral asaql ,vrou) r,uop nod
I 1,,;i.p.ro^;o
qJo noA iuooni 9,,) uraod aql Jeaq l,ueJ +Eql Suqlauos Jo auoaluos sassaJppe dlna,rp uraod
e uaq1vr sr (g) aqdo4sode uy 'asuas .radaap aruos ul an4 aq o+ lno surnl
]nq uorlrrpe4uoJ e
a{rT spunos 1eq1 Suqlauos sr (3) xopered v'spunos pDads aleru l,uop sarra;pd pue suad
1nq'(,,raqlqs,, ^?a) are daql leqm arrT prmos sprom uar{,vr sr (g) eradoleurouo .lp)rror{delaur
lou fl srql 1nq'raqloue Jo sural ur paqrrJsap sr 8try{l auo uaql\{ sl ,(v) roqdelaur y .8uorm
1sn[ are sJamsue rar{io aql ilV 'au]l e ur dypaleadar punos }ueuosuoJ lel}r-q aues aq1 Sursn
suearu qJr-q/vr'uoqe.ralqp 1o aldruexa ue sr ,,qsqod ol uad rnod;o sarra;1d aql,, aserqd aql
C'OI
'8uryr1a4s pue SurpearJalo aluos sarrnbar,,sla8ue,, ayqrr'uosrreduor
+JaJrp e s srad.uel
leql aas o] alqe aq p|nor{s nod'leql dq pasn;uoo araa,r nod Jr ua^g .arard ptrr4es E qrns Jo
a|pplru aql ur iou dpnelrar 'qa8ue ol sallaswaql a;eduror dleraua8 1,uop slaod pagaJuoJ
uala puv'dlarms Jo sraquaru se qaod lou'1daruo3 oq1 :drqao4 lnoqe 8uq1el sr arualuas
ler{+lng 'saserqd aql Suqeads qa8ue aq} are slaod 1eq1 dldurr lq8pu qJr-qm ,(99 aqD ,,qa8ue
;o aserqd dral aq+,, aq ol prcs sr d4aod asnpJaq dlorrl aFlll p sr ,qa8ue ,(g) 'srJqder8ou
-orqJ ueql Jailaq aJe slaod se 1sn[ uaursumo] uprn Ja+aq are sradrvrel :uoseal atu€s aq] JoJ
Suorm sr (q) 'parroJq sl (J) os 'uaql a{{ tou's.raqderSouorqr ueq} railaq aq o} pres are
spod pue'suerrolsrq plol aJe s.raqderSouor.lf, 'lnoqe alr.rzvr sraqderSouorqr Surqlaiuos 1sn[
aru daql laraq Surqldue o1 pareduro) uala ]ou are srodeu asneJaq Suorrvr sr (gj .uerrolsrq
ueql drolsrq rnod a1rr,u laod e aleq ol ragaq $ 1r ,os 1sn[ prre'(gg-lg satnf uerusu.,u.o1
_E
3ura11n1s e ueq+ ,,asneJ s,auo peald o1 rar{rvre1 lue8ala up alpq ol,, Ja}+aq sr 1i sdes aqspN
V'6
'luelalarrr 1sn[* (y) 's1aod
Jo lsrl sn0 dq pauqaqzvrralo 8ur.pa8 ara1vr nod;r nod asnluoJ o+ araql sr (q) .spueqsnq a^€q
sa^r.lvr asnpJaq'(l) Sulsooqc o1u nod ra)Jns l{8lu aJI.{{
Jo uorJou aqt,palzzeqdran 3uq1e3
ararvr nod I 'a8essed sF{l ul pauoquaru sr aqs pue 'q}eg tuo,r1 s,ar{s asneJaq'qleg
Jo aJIM
aql palleJ raJner{J uI ra}Jurer{J snoweJ e sr aJaql asneJaq Sursn;uoc aq lq8rur (g) ramsuy
'ua+snv aue{ ut oslE lnq rarner{J ur dluo lou dn sMoqs umo} ar{} 1nou1 o1 papadxa
1sn[ are
nod luql uoqsanb a8payvroul praua8 e $ srqJ 'pue18ug ur eds pue u^ro+ luarJue ue $ r0eg g'8
'qleg Jo aJIM aq] uo
{rn}s 1oB pue a8essed srql
Surpear aFq,rt ralo paze1? sada asoqrvr aldoad roy s1 (g) 'plalJ .aueu
ual Jo lno lq8re.4s sr (q)
aleuqel aql dq pa1sa88ns lnq paJJo)ur dldrurs sr (3) '(,,aureu uad,, suearu 11) sueaur awnld ap
M7u l/Jlqii' lvlouT or{lvr aldoad asnJuo) ir{8rur (g) '(oogt-r7g1) sepeg np a}snFs ap arunellrnD
se depol u1lroDl dFergure; aroru dgenlre '1aod qtuau e sr aq nod q1a1 snusnles arueu arD
^
Surzvrollo; dltcarrp aserqd IeJqar{luared aq1 lprervr,ropq8re4s &lard ,{y1en1re sr uoqsanb sHI
V 'l
'sau{ parJrrads aql;o Suuearu aql urory plaqe reJ Buqp8 s,}eql
}ng
'are1d lsrr; aqq ur auodrala lnoqe 3u1u*r are slaod aw dq,rt ur dn
lq8ner 1aB pue {u1lua^o
oqm asoql roj ldarxa 'a8Bssed aq] r{llm op oi 8urq1,,(ue aleq dlear l,usaop (g) .1noqe ,(rrom
o1 suoqelndal qlIM aldoad;o ,{ro8alet Jaqloue se idarxa slsrcq}e ur pa}saJa}ur
lou $ aqseN
1nq ,,'lrrap aql Jou poD JoI Jar{llau aJeJ +pr{+ asoql,, ol aJuaJaJaJ aq} uo snJoJ or{n aldoad
>1cr41q8rur (q) '8uurydn sr d4aod leql papl praua8 ar{l }ou'suorlelndar rraql roJ slpnphrpur
Jo suraJuor puosrad arp dlleJIIIJads sr a8essed sgl uI uraJuor ar{l tnq'd4aod dq parrdsrn
are srarplos leql uo4uaru saop aSessed aql asneraq 3u4dua1 s1 (3) (rpJ seM aq
{ultg t,up1p
sarrerodrualuo) snJ os'auqalT srq Surrnp JaJapuBIs e palaprsuoJ uaryo se.rr ar{seN,+q8noql
aleq dew aq leq,vr elrdsap puy) 'saut asaql rn ssaurreJ lnoqe pres sr 8tlq1ou lnq,(g) qum
aar8e 1q3nu aqseN 'suoqelndar Jlaql JoJ rea; dq pue 'aureqs rorror{ Jraq} Jo aruey .roy Bur
Jo
-3uo1rraql dq'uragl lnoq' pr€s sr r'q./vr dq pale,rqoru are aldoad ,nroi{ aqrrrsap gt-6€ saq-I v
"',#?:'#:iff ifi l;:;'5:TffiTiil:;1x;Hffi *aightrorwardsrammarquestion
12' A Choices (A)' (c)' (D)' and (E) are all defenses
that Nashe uses for why
respect' leaving (B)' which just poehy deserves
is repoffi on *ni.r, p-"pil;;;i.pr in awe by
13' A This is a pure definition question; you need poets,pens.
to know what a tautology
unnecessary repetition,so (A) is. A tautorogy is
and so seem to fit'but do
is the right urry:r..(u), (q,;;6iailsound like criticisms,
not paraphra'se the cited'lin"lif;);;t;
but sermo"s u'e,tof by definition says that the preachers
l,?J;.**""s' ineloquent, as siecified in
the sentence
14' E The selection specifically
mentions all other options
(B) is as functions of poetry: (A)
in tnes 54_Jz; (C) is in 1i"., 4&2;r"jtol is in line 40;
function of poetry but is i,
mentioned in the selection "u,
;ffiffi;om hne 30. (E) is not a
u ruil ir. or lor-g preachers.
rs. D Thf is a hard question asking for
general knowredge. you
courd ha
rearized that peopre n"":,r:1:F;""t"ili:
have led youlo thoo" the necessirv.or poerry
correct *r*u.Ipluto. (Actualry,
;:f;i,:T:1,il,:*'jilr'
reallv necessarv in his RE.ubtic.) irili"'a..ia.s that poets are-n,t
N";h; (;j ffi" ;r,#;i,ffi[:tion. sidney (B) and
rn*uonua i,,ihe te*t. Milton
#:r*:;$rffi,::1;fn*r G);th. ramous British poer

SUEST|0NS t5_28
john Donn
*,'",,*niffi.i"|r;:lifff xiiif,iil:,;,iff iiliii:!r,*,,:ry::#ff
subjects both sacreJ utta
ptoru*lq"b'"L9,;;#.;;""toriously HX";H"'ff %H;
figurative language, which airn.rrtl".ause of his ingeniously
is why he tends,rh"* ,o
This poem, "A valedictio"' on English
frequentty.
ori,vu.pir;; .;d;s:I"q:;o*;.*,"*u*, *rrated from one,s
shirts oipe*pu.tiu", phys with ihe;J,d";;,";i rover.
]lill:luf;1?if:,',:ff i?
presence and absence,
16' C rf vou are finding ir difficurl:o^r:1{:.tr:lv
answer this question' The yh:t ihir-pg.* is about, vou can use poE ro
easiest one to eliminate is
ing here for a pleasure .rt^.-:-S:_: @j, becau#;
is far too much cry_
tempting' but you should
;;Fi *;: ,:Td apocalyptic, so (D) might
have realized that,ti, o.u sound
world imagerv is meraphorical love poem and that the
end of the
lt*:J,."*l *"" tGffi;;it
metaphorical death' so (B) is
wrong' That leave! (A) and (q.
is probably arso a
ill;;; the reration_ro te' if
:$m: nruU"it:X":S**'.1* l"i,tu,,.., but the rou*lu," derinitery being
IT. D youshould know
the defin
Exam'ikes''"*ffi;;rl:i, j#iTir-^{{r,.::Lff
simile that occurs in a meiaphvrl.riil;. T:'f ffi fl IT:',:rT::,:y,.
cal poem' since it is by
v"I'r'n""ia;;;;;;d;ft this as a metaphysi_
JohnDtr,r,e. br,ect iJ,iJir.ru.y -ou"'n#t
Note that "metaphysicul" h.r. overview in chapter 2.
of meaning'
rtur rittr" to i" ,"rin il"i.r*pi;;^""1n'r.n
ro do with depth
metaptr^ttjT*..",u
tear becomes'tre
an entire world once +iull# isthe coirlarisii oi,"u* to
j* ;;;;; bails become globes
it reflects his beioved, grobes-each
once someone pastes images
of the continents on them: 7";;t#;
answer (C)' recognizing tiat been rempted to
this is r;rfi;;;rical, but
in this instance the lines are

2(l(l ; CRACKII{G THE AP EI,IGI.ISH


I.ITTRATURE IXAM
l0U I SN0lMVtdXt 0t{V SUlrltSt{V :Z lsll t)tI)VUd

nod pue'uraod aJqua aql lnoq8no.rql patnelsns uaaq spq dra8erur punor aql asneJaq
-
dlsnovrqo (l) 'sapp pw ,eas aq1 ,uooru aql uaam+aq uoqJarruo) snorlqo ue sr araql asneraq
Eq
aql sllJ ($ 'srarvrzue aw uI llJ +,usalp lpr{,vr roJ 8q1oo1 ,rn nod os ,aiaq rfunrrorrn
^nraod
Suotm aq dpuapl o1 8uroo1 a.re nod 1d1ryarur uoqianb iq1'pn", ol raqwauau .araq
tno
nod dlaq uer uorleu.ruIlH Jo ssaJord prrsdqdepur *ouurnroc aql dlrrer
{Q'ft+aod io qlrrvr
lIurBJ auros pue ruaod aq1;o uoqeprdralq qloq sarmbar lr asnpraq uoqsanb ,(rl;li ; q sF{l o .02

'8uorm sr (g) os ,asnep luapuadap aql dq peqryoru


lou eJe lnq ,,ueluTro1vr,,
pafqns aq1;o spa[qo aru quauquor palsr1 aq1 .8uo.rrrr. sr (61; os-,,qJrum,, fq pngroo*'e*rq
lou sr pahns aql lnq'asnelt luapuadapur sHl Jo loahns aql st uur4ro; .buorrra
s 131
os',rea1pazr_oqdepru e $ ll u€q,l ralel sarq ezuels sql uI ".I,l
dn,r,roqs
I€JaAas Iqlm ua1a lou
saop ,,plroM,, 'SuoJzvr sr (y) os 'Srmndue olul waql a{€ru lou saop
lnq-sardor aql ol sJaJar aq
ldoqs sFI u.r ,,dq qleq,, uerDIJoM aql slaporu ale
,,sardoJ,, aql 'qapow tuory pardbr rrn .nqo1d
aql q8noqllv 'sIIPq prmor aql ol raJal lmlu ,,qJnl1vr pw ,aro;araq1 .doqslrorvr
,, ,,lE:!{L,, rfloq
sp.l uI Tuplq arc +pql sileq plmoJ aql .uer1nlrollir
Jo lno saqoy8 Suxeru $ upru>lrom arll aql
sl ,/a{Bru,,;o pahns aqL/?Teru t, qtalaql;o pafqo aql fl
,,lprll ,,.s1 ,,+etq+,,Jalaleqa ol
sJaJaJ //rprq.{vt,, os ,,'+erll,,3urr{rrporr asnBIJ luapuadap e su€aq,,9JI{M,, ,uraod
sr-ql uI
arn+Jn4s aJualuas aql lno uos ue) nod qarvr rvroq aas o1 uoqsanb reuurerg e dlear q s1l[ g .ZZ

(s)a1u'.n4rs.s,a^oraq+uaa'+aq,.tr"",:I:ffiT"1',#$ll;:f:,:lillJ::i:,.{ililJtr
q (d 'urao{ aql lnoq8no.rql pauplsns Suraq uuql raqler 'ezvels puoras aql ur luaururord
-
dluo dpa.r sr dra8eun aqo13 aql 'a.rour.raqlrng 'ruaod aqiyo Burpear'apldruorur ue sr (g) Bu
-sootp os 'sreal lo sa.laqds dup aql tmlll,lr uraql Swureluor dq saJuelsrp
lsEA aql aJnpar ol
sI uraod aql yo lurod aloqm aql lnq'sacuelsrp lspl pue saqo18 ere arr,p ,srrnr"q,r,{gp
n".{
st (g) 'ssaussaladoqgo asuas E araql $ Jou'ssaussalllrorvr yo ,{.ra8eur due
irrrnr"q
1,usr
8uo.'vr(y) 'aldruexe up q sr-q+ qJr-qm 1o d4aod alol rn relndod dp'adsa"r.q1
s1 ,rprr,
saprynb asaql 4Fuap pue ssaualoqm paFad e+EJrp,o "pn*
pue pua ou a^€q salrrrJ
'adeqs pa;rad "ro;irrql
1{l se olrrlJ aql sla,rdralur d4aod drnlua>qluaalualas Jo-lerroleur prroprd
ro-dqderSouoJl atp asneJaq'dra8eun alJrrJ rneluoc suraod prrsiqdelatu f**,X*1a .17
3
,,'alaqds,, pJo,lr aql pue uooru aqt
seq ezuPls PJFD atf,],'splrolvl pue saqop s€q earels puoJas aq1 dem e ur drueu$a.rd uala
pue 'llruJ pue stnoJ seq ezuels lsrrl aq1 .ssaupunor yo sa$erur rneluoJ spzu€ls aql
Jo IIV g .02

-rorur aplrrur (a) pue'(d'b) 'parrorur aroJalaql $ pup'arroqc lsrg aql *qs {tiTl}l1td;
ul ua$ lrar.ror ale_ saJroqJ puoJas pue lsJq aql ,arolaraql .sJalen Jaqp ol 1o,r,rrffi
se
ol $ sarq snonard aql tuory Sumuguoc q leql uosrledruor aw pue,sauq oru; asaq] rr
paJualeJal dlerunads lou arp daql'uaod aql rn pauoquaru are *ei pne rn"r q3no.11[Vl8rrt
-qlolJ lnoqe areq 3rry11ou sI eJaqilaJpJ Jaq uo
,,uro/vt,, sreal s,pa^olaq aql osle aJe daql pue
'paropq srq;o a8eun aql ,,reaw,, Wrqaa ,srea1 s,raleads aq1 ale gI pue,I saql ul sJeal at{I g .6I

'(3) pos s,pod arp suonuaur uraod


sFIl ul Stmnou puv'srure ou serl ($ pFo,,vr aqJ'uooru aql (v) 10u dlsnor,r,qo g .**1ro^
aq+'(d ro (g) raqlra ol saJror{r rnod smorreu qJrqm ,(tZ aql) flure setl dlqua passarppe a{l
f
asneJaq sul panlear a^pq ppoqs no^ 'pa^olaq aql o+ ssaJppe up sr .gI
,,uooru ueql aroru o,, iI
aql alerrpur ot (?I aqD ,,rllop os,, Jo asn aql aloN :alnurs e lnq,roqde-"*;';;"*jJriT;
should remember that you have already been asked
about round imagery. Remember to
keep your answers consistent across questions. (E)
probably fits because the poem certainly
does describe.unhappy feelings. Thatleaves
*itil (A) and (D); you must-ciloose between
a goddess and the man in the moon. There is"r
no explicit refermce to either a goddess or the
man in the moory but at least the idea of a goddess
seems flattering to tn. u.toved and fits
the poem better. Therefore, (D) is the least fikely association
and the correct answer.
24' D "Diverse" here just means "diffetent";the phrase
means "a different place.,, Heaven (A) and
hell (B) have no place in the poem at all. Europe (C) is listed
as a contnent in the poem, but
not specified as a destination. The ground (E) is jusi an
answer for the truly Jesperate.
25' A Tears are found throughout all three stanzgl,.although
you have to look closely at the final
stanza to find the idea in the word "weep" (line zt)."Glfbes
(B) ur" oo1f i1tt*,".ora
stanza' coins (C) are only in the first stanza. The moon
(D) is only i" tri. tr,i.a stanza. The
ocean (E) is in the second and third stanzas. ocean
is almost suggested by the mention of
as fr"at of ,,i""p;, for tears in
shores in the first stanza,but it is not as r-fu"g a suggestion
the third stanza, so tears (A) remains the bestinsweli

26. B Line 4 is "And by this mintage they are


gomgthing worth.,,
,,They,,
refers back to ,,my tears,,
in line 2' The tears are said to be coined by the betved's
face (line 3), and the coin meta-
phor is carried forward into the use of "rnintage" here;
ttre process of being coined is the
mintage that makes the tears worth
gomethin{. The tears reflect her face thi *uy that coins
show a ruler's.face. By showing the beloved's face, the
tears become valuable, like coins.
Therefore, (B) is the best paraphrase. (A) is conhary
to the sense of the poem, because it says
the beloved is- worthless. (C) does not reflect.the
meaning of the line, some readers
might be confused if they don't know what ,,mintage,, "itn""gh because
nieans. (D) is tempti;g, it
sounds nice and fits with the meaning ofthe poem,"frt
it overstates-it goes beyond what
the line means. ut" ,uppor"d to ;estate, not extend. (E) is b;th;ague and
faraphrases
of New Age, and is not the point.
sort

27' C The speaker asks the lover to "forbearf To teach the


sea what it may do,, (lines 21-22) and
that she not let.the wind'Example lind" (line 24) in her
behavior. ri. i;,;yi"; rhat the
natural elements watghing and learning from her, copying her behavior,
ai idea that
ft:-
is carried forward-a1e ,h: description of iis lover ur tr"rl
-*" influencing the tides. One
would think, in the face of such great love, that she would
have the po*"rio ur"ak his heart
(A), but that is not mentioned in the specified lines.
He does mention dying in these lines
(B), but mostly in the context of her power
over the elements; he warns her against teaching
the oceans and winds how to kill him, not against killing
him herself. The lines assume that
she has the power to restrain her grief (o), b"ut just
tell hir to do so; they do not emphasize
this power the way thgl.do her p6wer over nature.
Nowhere in the poem is there anything
mentioned about the right to take other lovers (E).
28' A The speaker flatters.the beloved by exaggerating her
powers over natural elements, by de-
claring the inestimable value of even theleflection, oi h.,
image-in other words, through
hyperbole.

202 S CRACI(ING THE AP ENGLISH I,ITTRATURE EXAM


toz I Sll0llvt'MdXl 0NV SULT SNV :Z ISII l)tI)VUd

'ramsu€ alqlssod dp9 q (y)'aro;ararlJ'aslpJ sr luaualqs aql-$lrr4 due auodue qreal


",tl
ol3qd4 sI auo o5:Stmnduu uo lq8q Iear ou spaqs leql acelduoururoelueddrg e dprau sr
A 'an4 alotaJaql $ pu€ 'a41 urvro Jaq ol stro4Jeal s,€slno-I dq pue aldoadsumol aql Jo suo4
-Jear atn dq papoddns pue a8essed aql dq palsa88ns auaql € sl
AI 'ueruog Juarrup u€ se^
resaeJ l{3noq1 pue a8essed aql paurugs oq t aldoad asnJuor o1 araql sn11 .a8essed srul
rr IaruJ s dlanos 1eq1 uorlsaSSns ou sr araql'1uau4ea4 s,Jesae3 yo arrorddesip nod ua1a
Jr
asneJaq'a8essed aq+ of luplalal lou sr lnq'eapr Suqsaralu ue $ .anJ+ aloJaJaq+ sr pug
11
'a8essed aql Jo €apl rneru aql sassardxa I urall 'dlaleredas luarualels r{Jea arnru€xa ueJ aM
V'I€
'eJne arnlnJsetu aloru s,aof pue saIlIAIpe s,Eslnol uaamlaq
lse4uoJ qr dq uorlelardralur
srql slsaSSns a8essed aqJ 'sallllrlJe aurunual lleuourpe4 aleJrpw ol ursrJqrJJ
lsrunual gr
pasn dlaprrvr rural e sr qJrqa ,,'araqds lpJnllnJ au.n4ruaJ,, aserqd aq1 sasn (g)
1nq1 ,pp
:1g;
salPal srq|uMol aql Jo sanlel paq8rq aq+ +ou dlqeqord are daql 'saqrlule al4onpord-ssa1
raq Jo dueru dols raq a{uur IIIM pueqsnq pue rvtefur-raqpru Jaq leql saledrcque rqr rsnnrrq
lnq's1se1 Puoqlpe4 dueru u1 pa8e8ua fl esrnoT asn€raq'Suqdural s1 (g) 'ptpl due;o snruad
e $ aqs leril
lsaSSns o1 3rry11ou sr araq+ 'asnoq.raq Suruealc agq,t,r ,,lsqre ue Jo usersnqlua
aql lsourlB,, seq esrnoT qSnoqlp 'asnelaq Suorrur sl .ttt dgeluaru $ esrnoT
(d leril lsaShns
o1 Etmpou sI aJarp asneJaq 8uo,rm sr (3) '1,uare sarlrlr+Je dluapleru s,esrnoT
leqrvr'dlasnard
sI qJrqM'lenxas dlensn aJu sassappo8 rraql pue sFn+rJ lsalJetl'saprsag 'FnllJ
+salJeq
puoseas due aqorrtn ol ruaas l,usaop leql pu€ 'sqraq;o Sugpsrp s,esmo-I sr uoqf,alas
sHl ul Supurey ol aruaraJar u ot asolr sauor teql8ulw dpo aq1 :snolnrrprr lsourp sr (y)
'uorlJalas aql;o Surpear
lryareJ e pue gocl qSnorql uonsanb srw ramsue unr noj,*n3+ s '00

'aJrT parrreu
Jo saqlnJrJJrp aql
gul
-pre8a.r ruopsrlvr lEuoqualuor aql uI salartaq aqs ,luegodun aloru lnq llaqlour s,ao{ pue ,ao{
'J€sae) lnoqP plol $ aqs ]eqm salalTaq aqs se JeJosw +ua)ouut osle sr aqs pue !r srvroul pue
'parallaqs dluregar $ esmo'I'(d sl rar$up alqeldarre dpar dluo aql,arolaraq; .1am dialu
-raqlTap sI rolleqaq joq uI 3rry11ou puu '8op aql sa^ol osle Esrno1 1nq'resae3 ro; dqledruds
mo J^o asneJaq padde oqe lq8rur (g) ,{em qt ur'q8noua JqsrTear dlurepar sr a;u ratla$ueqo
lI,!' a8erlreu Suruordn Jaq moq;o uorlelduraluoJ Jaq 1nq,qs11oo; uaru lnoqe seapr s,ESrno.I
pqt +q8r-ul no^ 'lg dlear l,usaop ,,r{suooJ,, pue €qlqruar} JoJ suoseal Jo slol alp araql +nq
.'sa1qura4,, psmol Tt.{ ur a8essed aq} Jo pua aql ol alelal ol sruaas ,,paua1q81q,, asneJaq
€urn1e osle $ (3) '8u1p_earralo aq plnom lpql lnq,qspgrlr leq,/v\auos are uaur lnoqe suoq
-daruorerd rarl l'rfl an8re lq8pr no^'rlslpF{r BunDdue Br4op uaoqs +ou $ aqs ,alreu
lnq
il esmol asneJaq'lq8g dlped sr (g) rvreyu-Jar.&oru alqnJ Jaq lnoqe sreal raq rpr.u psrnoT
_ pasnJuoo nod aseJ uI alaql q u,lvroJtfi seM l€ql 1nq'Suuaauuop Jo Janrq.uaruom
:3uo.rzra, dlsnorlqo sr (v) 'goa asn u€J nod qrrum ur uoqsanb e osp sr sHl
lou $ esmo.I
lng +noqe
saddolarals relndod dueu o1 qeadde esmo1 yo lre4.rod alp asneraq notrranb d1Jr4 E sr sn{J
O.'62
'Ile ra$e paIJJ€ru pB o11uem_l,usaop dlpar aqs
lerp luaula8e8ua Suol drarr e rarye sappap oq6
rrEruo/vl € +noqe '169I uI paqqlqnd ,,'!rnN puelSug llaN
v,, d,ro1s goqs aql ruory sr uortralas srql
'8uqum raq ruo{ s8rmrrea raq q8norql aruapuadapur
JnuouoJa a^arqJe ol alge a1u4 Jaq Jo uatuolvr l\,ral aql Jo auo osF spllt ueruaaJc .aurq Jaq
Jo sJalrJM
puot8a-r rn Fnsmm rpdap pcrSoloqodsd e rago srapereqJ Jaq lnq'Ja1u^rvr p.rot8ar pue ptrear dpna
luepodun uE se palou dle.raua8 $ aqs 'sJlssep dreralq Jo spuarq qlrrvr suorssnJsrp rarl dq pacuangru4
dpuno;ord 1s_our lnq'palernpa-1am spm aqs 'arn+lm arussardar pue puoqrpe4 e uaruom uo slJoJJa
Jo
prrSoloqrdsd aql pade4rod leql uollrlJ alorm aqs'lppe ue sV'ploqasnoq Jnor8lar d1pr4s pue paqsl
-.razrodrur ue ur dn lq8norq s€rvr orl 4, ralrmr puelSull rvraN e ser'r (6961-7991) ueruJarg r"nftfn .g irnru

6t-6t sNotls]n0
32. C There was for many centuries a well-regarded theory that said personalities were influenced
by "humors" in body: Warm humors in the body caused u.g.y or passionate person-
the
alities, while cold humors caused unemotional or calm personalities. These humors were
affected by diet and by environment. This is where you get the stereotype of the hot Latin
lover who lives in a hopical regory eats spicy food, andls given to fits bf violent temper.
It is also where we qet the idea of people being "in a good humor." The bitten neighb^or
is described as "choleric," which is another reference to the theory of humors. Louisa is
deliberately feeding her dog bland food to discourage any furthei attacks on the neighbors.
She is not poor or she would not be able to have such nice things in her house, so (d) is
false. The passage does speak metaphorically of Caesar's impriionment, but Louisa is not
deliberately punishing her dog, certainly not for a decade, so p; is wrong. There are many
references to hermits and nuns in this story, but dogs do not practice cet6acy as a religious
practice, so (D) is wrong. (E) is wrong just from context, because the passage mentions that
other kitchens give bones to dogs.

33' B "Redolent" means "smelling of," andbecause purity doesn't have a smell, its use here must
be metaphorical. Irony (A) is tempting, but there is nothing opposed or contradictory here
to indicate irony. There is no simile (C) because there is no diiect comparison of two things.
Because there is no contradiction, there cannot be an oxymoron (D). Niothing is alluded tJ,
so (E) is wrong. If you do not know these terms, study the glossary in this b"ook.

34. A The story of Caesar is a gentle satire on the minidramas of small-town life, which finds
excitement in the vicious reputation of an old dog. Clues to the satirical tone are the many
overwritten references to sin and danger, and especially Louisa's vision of Caesar on a ram-
page through the town. Freeman's treatment of this passage is too humorous to be either
indignant (B) or pensive (C). Because it is clear to the reader, and even to
]oe, that Caesar
isn't really dangerous, there is nothing foreboding (D) or menacing (E) in this passage either.
The selection does say that Louisa feels many "forebodings of disturbance," includiig rvot-
ries about Caesar, but those are Louisa's feelings rather than the tone of the passage, iuhi.h
indicates the attitude of the author.

35. E "Sanguinary" means bloody, both in the sense of containing blood and of liking blood. (D),
vegetarian, is obviously wrong because it contradicts the meaning of the sentence. The other
answers draw on ideas raised in previous questions on this piece; remember to keep your
answers consistent. We have already established that financial concerns do not dominate
Caesar's diet, so (A) is wrong. (B) and (C) drag in the ideas of masculine and feminine traits
that predominate in this selection, but they really have nothing to do with what the dog
eats.

35. C This question testsfrow well you read Louisa's character, because the entire passage is about
her attitude toward gender relafions. Her meditations on the disorder her future husband
will bring to her house and on the impending danger of Caesar's release are best summed
up in (C). Louisa is living in a society that believes men and women belong together (A),
but her worries show that she is not entirely convinced of this. On the othei hind, she is not
explicitly rejecting marriage, so (B) is not the right answer either. (D) introduces the idea of
wildness that you might have associated with Caesar, but it is important to note that Louisa
does not believe she has tamed Caesar, nor does she think she wiil have any influence over
her husband, so (D) is wrong. Her belief that joe's decision to release Caesjr will prove di-
sastrous shows that she does not think men are more intelligent than women, so (l) is also
wrong.

204 J (RACKII{G THE AP TNGTISH LITERATURT TXAM


g0z I sll0tlvNv'tdxl 0Nv suil sNv :z Isil t)[]vud

'(6) reguns dleuuassa sr aJualorl IIe l€ql lugod s,tnum) aas ol nod peal pFoqs aplr
-oua8 rze111 qlrm qJnq)poorvr 8uryq yo uosrreduror,{ral aq1 luaod aql rn auol ol dlmqtsuas
e sannbar srw ftrzuSorar q8noqlp's;a11aq snorSgar ol relluns almb are sJallaq prullod 1eq1
'(f,)Jo rfin4 aql saleqsuoruap pzuels p4ql aql'(g) alermuga ol aleilsaq plnoM nod'aro;araq1
lursrurrvrreq IerJoS pue srzpN uaamlaq suoqJauuoJ uo JealJ +ou aJe nod p uana'tnm,req o1
aJuaraJar aq+ aJrpu p1nor{s nod'aswra4t1 '(y) alermuqa o} alellsarl plnom pue uraod aq}
14 JIJolarlJ slr.{l }o aruasard aq1 azruSoral p[no) nod'uorletmrua]xa rnurJal ]o JrrolaqJ IzeN
aql lnoqe ,l.ornl lou prp nod ua^fl 'anq alp sluaualels raqlo aql lo 11y'uraod aql to asuas
I
eJqua arn slJrpp4uo) (g) 'pel uI ramflre paJJoJ aql pu€ 'aqe; sr (g) 'aroyaraql 'qsad uapre8
Sugutmuqa se qJns 'r(ressarau se paqqsnf uago 8uryu uana '8tngq IIE suuapuoc uraod aql fl 'Z?

aqt pearsru oqrt aldoad gdural lqSlw'a;rl depdrara Jo salneq aql lq8g lsmu a.{vr

aql'(g) leqt alou 1aso1c uara lou are (f) pue'(d'(v)


'praua8 ool lnq Suqdual st'1ua1orl"U[tT
dlenuassa sr plrorvr aql leql eapr aql'(6) 'srauapru8 ueqrnqns lryaread aslmJaqlo uala'auo
-,{rala qq}l,rl arp uorlnJesrad pue aJuelorl ro1 puualod aql leql g utaod aql Jo aruaq} aql i1 'It
'ruaod aq1;o pafqns aqi
sr Jaqlrau 1nq'uapre8 aqi;o uouduJsap aql uI pl4ur ol awoJ deru (3) ursruerrela8a,r pue (g)
u8rsap adexpuel 'uaod aq11o pa[qns rrletu aql uBtI] raq]er IIEpp roultu e $ +I lnq'sauq pu
-g aql rn pauoquau (@ ulearp e sr aJaqJ 'pafqns IerolIT arll lou $ lnq'a)ualorrr lnoqe auaql
8ur,{traprm s}r saleurumlil pue 'uraod aql Jo }xa}qns aql saurp[os (V) slzeN ol aJuara]ar leuq
aqJ '$lJnqJpoom Jo uapre8 aql plr o+ slduralle s,raleads aql saqlrrsap dgera111 uraod aql fl '0t
'fupno7 uourallor ZL6I raq ruory $ ,,$[Jnqrpoo6,, 'aJII ueqrnqns alqe]ro]ruoJ
dJ1

1o'ate daql se qons'sqldap aql atnruexa ol spual oqm laod druroduraluoJ paralar e sr urun) aqxpntr
ns-0n sNot$ln0
'3op Sqr8edruer e Jo uorsrl s,Esrno.1 aopeqsaloJ
lourrpJ os pue 't{ent t{uu rn Jesae3 ol JaJaJ }ou saop ,,aleJllapul, 'a8essed aql Surpea,r-Jalo Jo
aldtuexa raqloue sr (g) '8uorm are (q) pue (f,) os'dppoqs arB s8u8uolaq sTq leql rou'asreor
dlercadsa sr ao{ leqt 1sa33ns o1 3tm11ou sr aJaW '8uorm sr (y) os 'a8essed aq+ Jo uraJuoJ
,{reunrd aqt lou sI Joaraql TJ€l ro dterllap 'ao{ pue esmol uaamlaq sa)uaraJJlp aql srapuod
praua8 rn a8essed srul q8noqllv 'suraruo) Fnxas raq ol palrauuor sl aJII raq ol Surrq dew
ao{ soeqr aql }noqe uJaJuoJ s,esrnoT }Eql sI araq lurod aqJ 'a}eJIIapuI are spepp I€nxas
rrqder8 lalecrlapur are salof d1n6 ,,'pnxas dlalerrdorddeut,, JoJ rusnuaqdna e st ,,alEJIIapuL, g '6€

'(d a*t u^rolllerus Io arqes e


sr aldoadsurvrol aql Io rorral palel;tn ar{l pue 1(3) a8ueqr asryar aldoad uaqm suaddeq }eq/vr
;o aldruexa ue sr 1q31d pes s,resaeJ la8edruer € uo resae) Jo uorsrl rar{ uI pa}e4suourap
are (g) sreal s,eslnol lasealar s,resae3 sa8rn aq uaqn' (y) ptlpe.ld pue pq{ aq o} umot{s
sr ao{'alpu8rl r€sa€J aql Jo Euaurqsqdurorce alqeJrpu aql aqlrJsap Euaua}els aq} [V iI 'B€
'a8essed aq+ peaJ-ralo l,uo6l 'swdap uappq ol anlJ e lou'alqdrnsap dlprervr
-ropq8re4s lsnf sr alqca[pe sr-ql 'saop eslnoT ueql dpuaraglp resae3 aas a/vr 1nq ,,'pa1seul,,
aq p1noJ aJuaq pu€ 'snorJrl sr resaeJ 1eq1 uopsaSSns aql sI araql asnp)aq aldoad auros
palsFrr azreq 1q3p (g) 'acey sueatu ,,a?esqy,, 'pa8esr,r-ppu sr aq lare; altua8 e s€r{ rcsae) g 'l€
43' c "Darwinian pieties" is an- oxlimoron
because it is a contradiction in terms.
tionary theories are seen by many to Darvrrin,s evolu-
be antireligio,rri...rr. they conhadict
creation in the Bible' At td same time, "p!"ry" the story of
ir"un, ..tigious dLvotion. K;-* is suggest-
ing that people are devoted to Darwin
ur utinaty ur-in"y ever were to religion.
Darwin is used here not so much to suggesf Howevel,
.rrolrrtion as to suggest the political
associated wirh "social Darwinism,,- beliefs
wirich ,.u, **gg", b.#.";;;ilJor
violent zero-sum contest for survival in puoptu u, u
which the r,iinl"., are proven to be biologically
is orten a polite mask ror racism ani other social biases ugiir,rt
;rffi"Sj:i*,?:il#::il
M' D The tone of the poem contradicts the
speaker's attifudes, so nothing in the
those attitudes correct. Ail the other poem proves
statements about rrne 4 are true.
45' c The entire poem is an ironic satire against
people who refuse to recognize the
their lives as part of the violence of ihe violence of
*orta.'1a; *i'1o; ur. tempting because both
min and presumably the reader feel outrage Ku_
to protest it, but the poem itself remains
*.r *," uiolence in the world and would wish
irinic. There is
stanza,but its effectivenes, i, ur,J"r.rt
There is no helpless sorrow (B) anywhere
by the pathetic,:i:ffi.:if;ilflirlt]# ffifJ.^
po"J'
i" thi
45' D The entire point.oj th9 poem is the
speaker's growing awareness of his
tendencies' The idea thit all violence or her own violent
is relatei @) is"impried in the poem,
of the final two lines. The other uru*.r,
but is not the
ill|"tfft ur*ll variouslyirn"I"* r"rrreadings of

47' B The garden, usually considered a peaceful


place, is here the scene of violence,
ic' so choice I istrue' The garden ouuio.r"ty which is iron_
,y-forJ.r which is the site of Darwin-
ian struggle' so II is true' I-n this poem, "u*r.,
the garden is very realistic and specific, suggesting
that the issues discussed u.. ,p..ifi.
una ..Xnru. u, rv.il, so IV is true. Iiem
tempting because gardens are often feafured III is false, but
inpastorJpoems.Ilowevel, pastoral poems
focus on the peacefulness of the countryside,
which this poem obviously does not.
obviously false, and contradicts the maln Item V is
theme of tf,. po._.
48' B In the final line of the poem, "Nazi"
is an allusion to the atrocities of genocide
II' The poem as a whoie implies a comparison in world war
cide' but this is merely impiiea and
between the killing ;;J;h;cks and geno-
dols not become an outright simile "f (C) or
in the final line' Neither does the final line metaphor (A)
contain u putuao* @) or metonym (E).
not know what these terms mean, you If you do
should .onsuli the gtossary at the back
of this book.
49' c "Beheading the carrots" is an-example
of personification because it applies
a physical
man qualiry to an inhuman object: Larrots
ao",t t urr. t.ads to lop jicr6ing hu- the
woodchucks' actions in this wiy makes
the woodch,r.k, ,ourrd more "if;;;
sinister. Anthropomor_
phism (D) is a more involved form of personific"ri"tt
tions to the nonhuman
i" *nich one ascribes human motiva-
object.
50' c The speaker of a poem is not necessarily
the poet, but is a narrator. That makes ,,the
(A) the wrong answer. The other answers poet,,
are pretty ridiculous.

206 f CRACKII{G THE AP ENGI.ISH TITERATURE


EXAM
I0Z f SN0ltVNvldXl 0NV SUl/llsNV :Z lstl t)tl)vUd

//'uaasun/tueaJp//
ptrr- ,daap fsdaa1,, aseJ sryl ur 'sarudq,r asnardur a.le saurdqr iuelg ,(pca;rad siry rlJIr{M ,(g)
dluo saleal leqg ('urexa arll roJ ,vrou) ol ur,ra1 poo8 raqloue s,+r-,,lear Sururaas;o ftqenb
aql,, sueatu apnllF-lx]slJaT$ 'pauopueqe ]ou sr ruaod aql Jo apnllTnursrJa^ aql aJoJararll pue
'1ealrns uel.Il JaqlpJ JIISITBaJ sI pauoquaru ruealp aql asneraq Suo.rzvr sr (g) 'ursrrcluauruoJ
-IAua lnoqe 1ou'arualorl uerum{ +noqe sI lurod proru 3uo4s s,ruaod aq} asneJaq Suoral, sr
(@ 'ulaod arll Jo pua aql le uol+Jlpe4uoJ ou sr araql asneJaq Suo.rrvr sr (3) 'cruorr dlurepar
are sauq Fug aql asn€Jaq Suorzvr q (v) 'go.I qSnorql paraMsue aq ueJ uorlsanb srq; 't9
s
'und e q,,1q8p4u,,1eq1nod plol dpearp p uorlsanb dgeluapoul
2ooq
slql Jo {r€q aq} ur dressolS eq1 dprys o} arns aq,srural asarg .trouT },uop nod .(g) uorou
I
-dxo ue 1ou s,ll os !1aqr +Jlperluor lou saop +I 'uorsnlp ue se TJoM o1 q8noua rrprads dgear
lou sr lr 1nq'rualsds p8al aql o1 (q) uorsnlle ue se pear aq +qgpr iI .(g) Surq+due azqoqurds
lou saop ll puu'(y) .roqdelaru e aq ol q8noua paueisns +ou sr lJ's1JnqJpoom aql Sursse8
/$Irnqrpoom aql Suqncaxa ur pagusnf
;o ssarord prrsdqd aql ol Sur.rra;ar osle +nq i1a;
raleads aql teril ,vror{s o+,,aser lq8rpre,,ue lo eapr p8al aql ol Srnrra;ar,rmd e sr,,1q8r14y,,
J '99

'sluaruatels anrl aql qsq dprarror +eqt arror{r dpo aql sr (q) .spnqrpoom
uo snJoJ plnoqs ureod srql lo dreurums ou pue 'arnleu uerunq +noqe Jo pealsrn s)Jnr{Jpoorvt
+noqe aq ol suaas flns aql ]srrl aqJ 'ruaod aq1;o dreununs e paraprsuo) aq ol uoqerurolur
q8noua arrr8 1ou saop atnl +srrJ aq+ asnuJaq'aqey sr A 'an4 sr AJ os 'paueddeq dlear leqrvt;o
luaualelsJaprm ue q ,,1q31,r lno uJnl l,uplp,, +eql lvrou)I nod'ruaod aql Jo pua aq1 o11a3 nod
I
aunl aql dg 'an.4 osle $ pue luaua]e]s se 8tryp aru€s ar{l dpcexa lsorup des o1 asrnorsrp
Jnuape)e alol.u e sasn 11I ]uaura+els ZsaJn+eaJJ Surlq Sursse8 are nod uaq.,vr 8uor,u sr ]Eq1vr
1nq,,€uo.rn, +uam s8qql lBt{+ saudur ,,lL13g lno rnnl l,upIp,,1an.4 sr II iuarua}els .$lrnr{J
-poom atll Sursse8 lnoqe drols E reaq ol Suro8 ale nod 1eq1 nod sllal auq lsrr; aql 1an4 sr 1
luauale+s 'asleJ Jo an4 se +uarualels qJea al€nle^a 01 $ uoqsanb snfi JaMsue oi dem aq1 o'29
'pasryuoJ l,usr raleads aql pue ssalasuas l,uq aurT
aql asneJaq Suo.rrvr lno +elJ 1sn[ s (g) 'craodoleruouo dgenadsa 1ou sr 1r 'anrleral1le q aql ar{l
q8noqlp asneJaq Suo.rrvr sr (q) 'sauq snor,rard rr arolaq auo8 a,req sprom qsreq dlenba 1arn1
sIIl ol anblm lou sr paJJa aq+ +nq'pron .realc dlqsrer{ e sr ,,rallD{,, asneJaq ,an4 dlplped sr
(l) 'lqod srql dlpexa lswe8e Surn€re $ aloqtvr e se uaod aq+ lnq'd.rolepard dlernleu ore
sueumq l€ql splor{ rusnn.trreq proos 1d8o1oapr Jo aruanlJul arp Jo asneraq Suqdrual sr (g)
'sarSoloapr IZeN
Jo aJuanlJul arp tn se 'ayqeldarre sarlrJo4e Japual os pue dlanos uerunq rn
srrorpe plruq dueur azqemleu sldurs q)ns tErD uo4saS8ns E ua^a sr araqJ dplnrq os pe ol
ra>1eads aql Sutcuangtn ldrlrs IErnlTnJ I€rJrJqrE aql Suzrser{drua sr laod aq1 'auo)s pa8els e

Io eapr aql ol suapre8 prnleu;o 3u41as e ruory Stqnplrms dg dpr4 alltl e s uoqsanb sIqJ V 'Ig
HOW TO SCORE PRACTICE
TEST 2
Section f: Mulfipfe Choice

* _*______)
number correct number wrong Multiple-Choice Score
Secfion fl: Free Response
(See if you can find a teacher or
classmate tro score your essays using
+ + the guidelines in Chapter
Z .)
essay ------
1
(out of 9)
essay 2
(out of 9) .
essay 3 Free-Response Score
(out of 9)

(omposile Score

x 1.23 =
Multiple-Choice Score
Weighted Section I Score

3.06 =
Free-Response Score
Weighted Section II Score

Weighted Section I Score


Weighted Section I Score Composite Score
AP Grode Conversion

208 I CRACKING IHE AP EiIGI.ISH


TITERATURE EXAM
rfuossolC

Potrebbero piacerti anche